Sie sind auf Seite 1von 70

SOAL TRY OUT UNIBRAW JANUARI 2008 1.

Male, 67yo complain of sudeen onset of decreased vision, metamorphosia, and paracentral scotoma. Clinically, we found elevation of the RPE, intraretinal fluid, subretinal blood, cystoid macular edema, and graygreen CNV elevation. The possible dx: a. NPDR+CSME b. Retinal angiomatous proliferation c. Retinal macroaneurisma d. Neovascular AMD. e. Polypoidal Choroidal Vasculopathy 2. If in the patient (no1) there is intraretinal haemorrhage at macular area, it may be an early sign of : a. PDR b. Retinal angiomatous proliferation. c. Macular BRVO d. Neovascular AMD e. Polypoidal Choroidal Vasculopathy 3. If in case no 1also found serosanguinous RPE detachment with multiple orange nodule, the possible dx is : a. Retinal angiomatous proliferation b. Neovascular AMD c. CSCR d. VKH syndrome e. Polypoidal Choroidal Vasculopathy. 4. For decision making process in the previous case (no 2&3), the gold standard is : a. fundus photography b. FFA c. ICG. d. OCT e. Multifocal ERG 5. Pria 43 tahun, dikonsulkan oleh dokter kulit karena akan memulai terpi klorokuin. Status oftalmologi yang harus diperiksa selain penampakan makula adalah : a. Tes penglihatan warna, amsler grid b. Tes penglihatan warna, amsler grid, fundus fotografi. c. Tes penglihatan warna, amsler grid, fundus fotografi, static perimetri. d. Tes penglihatan warna, amsler grid, static perimetri e. Tes penglihatan warna, amsler grid, fundus fotografi, ERG 6. Laki2 usia 62 tahun, mengeluh penglihatan mata kirinya mendadak turun sejak 12 hari yl, tanpa adanya nyeri. Visus OD 6/6, segmen anterior dalam batas normal, pada OS didapatkan gambaran n.optikus batas kabur dengan elevasi 2 D, hiperemi, perdarahan peripapil flameshaped. Pemeriksaan penunjang apakah yang diperlukan : a. Automated perimetri, head MRI, konsul gigi mulut. b. Automated perimetri, head CT scan c. Goldman perimetri, konsul gigi mulut d. Goldman perimetri, gula darah puasa dan postprandial e. Konfrontasi tes dan head MRI

7. Bila didapatkan skotoma sekosentral pada hasil pemeriksaan lapang pandang mata kiri, maka pemeriksaan yang diperlukan sebelum melakukan terapi adalah : a. Profil lipid dan hemoreologi b. MRA c. Foto thorak AP lateral d. Gula darah puasa dan postprandial. e. Analisa CSF 8. Wanita 35 tahun dengan obesitas mengeluh nyeri kepala yang berat, kadang melihat dobel, tajam penglihatan kedua mata 6/6. pernyataan yang sesuai adalah : a. Pungsi lumbar harus segera dilakukan b. Penglihatan dobel terjadi karena keterlibatan N III, IV, VI c. Keadaan spt ini bisa terjadi pada kehamilan, obesitas, pemakaian tetrasiklin, kontrasepsi oral. d. Kelainan lap pandang yang didapt berupa skotoma parasentral e. Tidak didapatkan kelainan reflek pupil 9. A patient come to clinic with severe panuveitis, the previouslyVA is 20/20 and now hand movement. The condition is not improve with cortosteroid treatment. The most appropriate additional treatment which properly given to this patient is : a. Cyclophosphamide b. Chlorambucil c. Methotrexate. d. Azathioprine e. Cyclosporine 10. A patient 35 yo, presents with bilateral redness, photophobia, discomfort of the eye and floater. Anterior chamber cell and pars plana snowbank was found in examination. Periferal retinal phlebitis with retinal venous sheating wasa also observed. Which of the following statement is not true for the disease : a. Assosiated with HLA DR2 b. Often bilateral, but sometimes can be unilateral c. Major cause of blindness is abltio retina. d. Usually the cause is unknown e. More than 50% is recurrent 11. A patient presents with retinal haemorage, microaneurisma, and cotton wool spot on the retina. Blood examination shows CD4 count is 40 cells/mm3. Which of the following is not correct : a. The retinal infection is commonly caused by CMV b. Cotton wool spot are the result of microvasculopathy c. Very early CMV retinitis commonly resmble as retinal haemorrhage. d. Ganciclovir is one of drug of choice e. Ganciclovir can be given orally 12. After underwent cataract surgery, patients visits the clinic with decrease vision, keartic precipitate, minimal hypopion and lens fragmen is found at antyerior chamber. Which of the following is not correct for the disease : a. The inflammation is caused by autoimmune reaction to the lens protein b. The reaction to the lens protein always granulomatous type.

c. The treatment is topical and systemic steroid d. Surgical removal of the lens is usually curative e. Intraocular pressure usually elevated 13. 12 year old boy presents with itching, blepharospasme, photophobia, blurred vision and copious mucoid discharge. Cobble stone is in upper tarsus. Theres family history of atopy. Which of the following is not correct : a. Usually persist year around ion tropical climate. b. Involve Hypersensitivity reaction type II c. Horne trantas dotes are developed in limbal type d. Antihistamine is successful to treat mild case e. Steroid pulse tx is recommended for exacerbation condition 14. A women 27 yo, presents with severe progressive ocular pain and photophobia. She use contact lens during daily activity. From examination, dendritic epithelial lesion and grey white superficial non suppurative stromal infection is observed. Radial perineuritis was also found. The disease shown no response to variety topical antimicrobial/antiviral agent or steroid. Which of the following is not correct for the disease : a. Diaminides is recommended for the treatment b. Giemsa staining can be use to visualized the cause of the disease c. The organism can be killed by freezing. d. Imidazoles is also drug choice of treatment e. In that patient, 6-12 month treatment is needed. 15. Seorang anak berumur 2 tahun dating dengan keluhan dari ibunya berupa : takut sinar; mata nrocos dan rewel/ Pada pemeriksaan ditemukan : Visus sukar evaluasi; CVI dan PCVI ringan ; photophobia dan lacrimasi. TIO digital ++ tanpa esotropia ataupun leukokoria. Pemeriksaan yg harus dilakukan pada penderita ini untuk menegakkan diagnosa adalah : a. Aplanasi tonometri. b. Pemeriksaan dengan lempeng plasido c. Sensibilitas kornea d. Fluoresin test e. Visual field 16. Suspect diagnosa penderita ini : a. Retinoblastoma b. Glaucoma congenital. c. Keratokonus d. Keratoglobus e. Staphiloma kornea 17. Penanganan penderita ini adalah : a. Enukleasi b. Eviscerasi c. Gonioskopi d. Goniotomi. e. Iridektomi

18. One child of 10 yo suffered of vernal conjunctivitis for 5 years. He preferred to used maxitrol eyedrops because it can release his itching and redness. Since one year he feels difficulty while he rides his bicycle. The possible diagnosis of this patien t are : 1. open angle glaucoma 2. close angle glaucoma .. 3. drug induced glaucoma 4. congenital glaucoma 19. The management of this patint (case number 18) are : 1. stoped the maxitrol eye drops 2. anti inflammation orally 3. anti glaucoma drug A 4. antibiotic eye drops. 20. 25 yo patient visits the hospital with blurred vision. Visual actuity ODS is 20/50. The patient also suffered from brochial asthma and often using corticosteroid nasal spray during asthma attack. Cataract of the both lenses is found during eye examination. What is the type of the cataract in this patiens : a. Nuclear cataract b. PSC. c. Lemellar cataract d. Capsular cataract e. Cortical cataract 21. A patient come to clinic with complain of red and pain of the right eye. The eye underwent cataract surgery a week ago. From examination, retained cataract material is found in anterior chamber. Cells flare and keratic precipitate is also present. IOP is 35 mmhg. What is the definitive therapy for the patient : a. Anti glaucoma medication b. Lens material extraction. c. Steroid topical d. Steroid systemic e. Non steroid anti inflammation 22. A patient visits the hospital with complain of abrupt onset of pain and redness of the left eye. The eye also blurred since 3 years ago. Eye examination show corneal edema and flare of anterior chamber. Lens is hyper mature. IOP is 40mmHg. The angle is open and precipitate of white flocular material is found at the angle. What is the most possible diagnose of the patient : a. Lens particle glaucoma b. Phacomorphic glaucoma c. Phacolytic glaucoma. d. Glaucomfleeken 23. A patient is under going preparation for cataract surgery. The axial length is 23 mm, average of keratometry reading is 40D and a constant for the lens is 118. The proper IOL power for this patient if calculated with SRK I is : a. 24.50D. b. 22.00D c. 23.50D d. 23.00D

e. 21.50D 24. After cataract surgery, ocular hypertension is found in a patient. Seidel test shows minor leakage of aquous at the wound site, although adequate and secure wound apposition is observed. What is the most proper option to manage this patient : a. Give pilocarpine eye drop b. Hard contact lens c. Avoiding steroid. d. Re suture of the wound e. No management is needed 25. Datang seorang ibu, usia 70thn, dengan keluhan benjolan pada kelopak mata kiri atas bagian tengah. Pada pemeriksaan benjolan kemerahan dan melekat kuat pada jaringan dibawahnya berdiameter 7 mm. Untuk menegakkan diagnosa, perlu dilakukan tindakan sbb : a. Excisional biopsy b. Incisional biopsy. c. Fine needle aspiration biopsy d. Wide excisic dengan vriscoupe e. Wide exicsic dan rekonstruksi mata 26. Pada soal di atas ternyata suatu keganasan adnexa, yaitu suatu Sebaceous Carcinoma. Atas dasar itu direncanakan : a. Wide excisi dengan insisi 3-5 mm dari batas undulasi tumor b. Wide excisi dengan insisi 1-3 mm dari batas undulasi tumor. c. Wide excisi dengan insisi 5-8 mm dari batas undulasi tumor d. Wide excisi dengan insisi 8-10 mm dari batas undulasi tumor e. Wide excisi dengan insisi 10-15 mm dari batas undulasi tumor 27. Penderita wanita usia 45 tahun datang dengan keluhan mata kiri menonjol secara perlahan sejak 1 tahun yl. Penonjolan mata kiri secara axial dan tidak ada rasa sakit. Tajam penglihatan pada mata kitri 6/60 dengan koreksi, sedangkan mata kanan 6/6. segmen anterior ODS tenang dan fundus kedua mata masih dalam batas normal, tapi terdapat RAPD + OS. Penderita mempunyai riwayat menggunakan kontrasepsi pil dan suntik selama 15 tahun. Pemeriksaan penunjang dalam pengelolaan penyakit untuk mengetahui lokasi dan perkiraan jenis tumor adalah : a. Hemangioma konjungtiva b. Sclerectasia c. Basalioma. d. Melanoma maligna e. Haemorragic nevus 28. Pemeriksaan diatas mengarah suatu meningioma. Pengukuran hertel didapatkan proptosis ringan pada mata kiri. Tidak didapatkan corneal exposure. Pengobatan pada penderita ini adalah : a. Eksenterasi b. Double enukleasi c. Orbitotomi lateral. d. Observasi e. Pemberian steroid 29. Tatalaksana awal penderita pseudotumor adalah: a. Pemberian steroid dosis tunggal 1-2 mg / kgBB / hari selama 14 hari.

b. c. d. e.

Biopsi insisi Eksisi tumor FNAB Radiasi

30. Wanita 20 tahun datang ke poiliklinik dengan keluhan kelopak mata atas kiri lebih turun. Keluhan ini dirasakan sejak penderita lahir dan tidak bertambah berat. Pandangan mata kanan dan kiri tidak terganggu. Pemeriksaan apa yang saudara lakukan utnuk menentukan jenis tindakan yang spesifik a. Margin reflex distance 1 b. Margin reflex distance 2 c. Levator Function. d. Upper eyelid creased e. Vertical interpalpebral fissure height 31. Penyebab tersering ptosis kongenital pada kasus diatas a. Myogenic. b. Aponeurotic c. Neurogenic d. Mechanical e. Traumatic 32. Pada ptosis kongenital paling sering dilakukan tindakan operasi 1. Mullers muscle resection 2. Levator resection 3. Fasanella servat procedure 4. Frontlis suspension surgery. 33. Komplikasi tersering operasi blefaroptosis : a. Wound Dehiscence b. Tarsal eversion. c. Over correction d. Under correction e. Scaring 34. Seorang anak perempuan berumur 8 tahun datang dengan keluhan kedua mata gatal dan merah. Dari hasil pemeriksaan didapatkan tajam penglihatan OD 5/6 dan OS 5/7.5, pada eversi kelopak mata didapatkan papil-papil yang besar dan injeksi konjungtiva. Diagnosa pasien ini adalah : a. Blepharitis b. Uveitis anterior c. Dacryostenosis d. Konjuntivitis vernal. e. Keratitis 35. Pasien diatas kontrol ke poliklinik mata setelah 1 athun kemudian sengan keluhan mata kabur sejak 1 minggu yl. Dari anamnesa diketahui bahwa keluhan mata merah dan gatal masih sering hilang timbul dan pasien tidak pernah kontrol. Pada pemeriksaan lampu celah yang didapatkan adalah : 1. Defek epitel berbentuk oval dengan kekeruhan di stromal pada kornea di bagian sentral

2. Punktata epitel erosi pada kornea superior 3. Folikel pada limbus kornea 4. Simblepharon konjungtiva B 36. Seorang bayi laki-laki umur 8 bulan pada pemeriksaan didapatkan epifora, fotofobia, blefasrospame, dan edema kornea. Diagnosa kemungkinan paling besar adalah : a. Epidemic keratoconjunctivitis b. Uveitis anterior ( Juvenile Rheumatoid Arhritis) c. Glaukoma kongenital primer. d. Keratitis bakterial e. Obstruksi duktus nasolakrimal 37. Seorang bayi laki-laki umru 3 bulan dibawa orang tuanya dengan keluhan mata putih yang baru disadari orang tuanya. Dari hasil pemeriksaan funduskopi didapatkan adanya gambaran persistent hyperplasic primary vitreous (PHPV). Hasil temuan ini bisa berhubungan dengan : 1. Glaukoma 2. Mikropthalmia 3. Hipoplasia Fovea 4. Ablasio retina E 38. Seorang anak perempuan berumur 2 tahun datang ke poliklinik dengan diantar kedua orang tuanya, dengan keluhan pada mata kanan menonjol, teradapat warna keputihan disertai mata merah dan anak sering menangis dan rewel. Dari hasil gambaran USG didapatkan echo high spike dangen kalsifikasi di dalamnya. Diagnosa yang paling mungkin : a. Katarak kongenital b. Glaukoma kongenital c. Endoftalmitis d. Retinoblastoma. e. Panuveitis 39. Anak laki-laki 10 tahun datang dengan keluhan juling, tidak terus menerus dan hanya muncul jika penderita tidak konsentrasi dan melamun, dan juling bergantian. Dari hasil pemeriksaan diadaptkan eksotropia intermitten tipe basic dengan kontrol yang baik. Penatalaksanaan penderita ini berupa : 1. Reses otot rektus lateral 2. Pemakaian lensa minus 3. Resek otot rektus medial 4. Terapi push-up C 40. Tindakan operatif bedah strabismus dapat menimbulkan komplikasi : 1. Perforasi sklera 2. Diplopia 3. Conjuctival inclusion cyst 4. Dellen E

STRABISMUS 1. A 7 month old infant boy demonstrates the ocular deviation is esotropia. Each eye is able to abduct normally. The child was born after a normal, full term pregnancy by a normal delivery. Considering the most likely diagnosis, the following statements is true : a. Vertical deviations are commonly associated b. Amblyopia isnt commonly associated c. Abduction movements often symmetric d. Adduction movements often symmetric e. Patients with this condition rarely require glasses during childhood 2. A mother complains that her 6 year old daughter has had increased wandering of her left eye over the past year. Your examination notes an intermittent exotropia of 35 prism diopters with fixation at distance and 5 prism diopters of exophoria with ixaton at near. Her uncorrected visual acuity is 20/20 OD and 20/20 OS. After discussion with her mother, you decide on surgical treatment. Which of the following tests would be least helpful in surgical management of this patient a. 30 minute occlusion test b. + 3.00 lens test c. + 5.00 lens test d. Worth four dot test e. Cycloplegic refraction After a 6 mm bilateral lateral rectus recession for an intermitten exotropia of 25 prism diopter, the patient develops an esotropia of 40 with constant diplopia and inability to abduct OD beyond the midline. The most likely cause is : a. Muscle slippage b. Accomodation spasm c. Surgical increase in AC/A ratio d. Inadvertent trauma to right sixth nerve during surgery A 4 year old child with horizontal nystagmus sees 20/50 whwn her head is held straight but can see 20/25 by adopting a 45 left head turn. The nystagmus is unchanged when fixing on a near target. Which surgical approach would be most likely to reduce her head turn ? a. Recess the right medial rectus and the left lateral rectus mucles Resect the right lateral rectus and the left medial rectus muscles. b. Recess both medial rectus muscles and resect both left lateral rectus mucles c. Recess the left medial rectus and the right lateral rectus mucles Resect the left lateral rectus and the right medial rectus muscles. d. Recess the left medial rectus muscles and resect the left lateral rectus mucles e. Recess the right lateral rectus and the left medial rectus muscles. A 5 year old has a visual acuity of 20/20 OD and 20/200 OS. Motility examination reveals a 6 esotropia at both distance and near. The 4 base out prism test over the right eye is most likely to show : a. No shift of the left eye b. Slow shift of the left eye temporally, followed by a fast convergence movement nasally c. A fast shift of the left eye temporally, followed by a slow convergence movement nasally d. A fast temoral shift of the left eye without refixation movement

3.

4.

5.

6.

A 25 year old male accountant sustained a head injury as a result of an automobile accident 9 monhs previously. This patients chief complaint is double vision when reading. There is no deviation in primary gaze, but a 15 prism diopter esotropia is present in down gaze. There is a left hypertropia of 8 prism diopters in right gaze and a right hypertropia of 10 prism diopters in left gaze. Excyclotorsion of 6 is present in primary gaze, increasing to 15 in down gaze. Ocular versions demonstrate underaction of both superior oblique muscles. What is the best treatment option for this patien? a. Reading glasses with prisms b. Bilateral superior oblique tucks c. Bilateral inferior oblique weakening procedures d. Advancement of the anterior fibers of both superior oblique muscles e. Advancement of the anterior fibers of both inferior oblique muscles A 44 old man sustained closed head trauma and now complains of object appearing tilted. The degree of tilting can be quantified by which of the following test? a. Simultaneous prism-cover test b. Careful analysis of ductions and versions together c. Lateroversion reflex test d. Double Maddox rod test Which of the following can not be used to test for ARC? a. Worth four dot test b. Major amblyoscope test c Titmus stereo test d. Cuppers monocular afterimage test A 6 month old girl with esotropia in a healthy child is generally recommended a. When the child is over 2 years old so that inferior oblique overaction and dissociated vertical deviation can be treated simultaneously b. When the child start to walk c. When amblyopia has been treated and deviation is of stable magnitude d. When the child is 4 month old so as to maximize the chance for developing stereopsis A 3 year old boy with a histry of congenital esotropia surgically corrected at age 12 months has visual acuity of 20/30 with both eyes open, but only 20/60 whwn each eye is tested monocularly using an occluder. The most likely reason for this discrepancy is a. Bilateral amblyopia b. Latent nystagmus c. Crowding phenomenon d. Binocular summation

7.

8.

9.

10.

TUMOR
1. A patiens 40 year-old with The malignant lesions most frequently affecting the eyelids are : 1. Basal cell carcinoma 2. Squamous cell carsinoma 3. Sebaceous cell carsinoma 4. Melanoma

2. A 1 year-old presents with a round, well-demarcated mass at the supero temporal rim. The lession has been present since birth. The most lakely diagnosis is a. Rhabdomyosarcoma b. Neurofibroma c. Dermoid cyst d. Capillary hemangioma e. Metastatic Ewing sarcoma 3. A 65 year-old women presents with a progressively enlarging mass in the right inferior orbit. Distraction of the lower eyelid reveals a Salmon patch appearance to the fornix. The most lakely diagnosis is a. Reactive lympoid hiperplasia b. Sebaceous carcinoma c. Melanoma d. Limfoma e. Apocrine hidrosarcoma Which of following signs is most likely to the present in patient with Graves ophthalmopathy a. Exopthalmus b. External ophthalmoplegia c. eyelid retraction d. optic neuropathy A 20 year-old man is struck over the right eye, and radiography show a fracture of the right orbital floor. Forced traction testing is equivocal because of poor cooperation. Four day after injury, 3 mm of the right exophthalmos is present , and movement of the eye are restricted in up gaze, down gaze, and horizontal gaze. Treatment at that time should be a. Conjungtival incision throught the inferior fornix, with examination of the fracture b. caldwell-luc incision and packing of the maxillarry sinus c. Skin incision over the inferior orbital rim and covering of the fracture defect with a plastic plate d. Skin incision beneath the eyelashes and covering of the fracture defect with a plastic plate e. None of the above

4.

5.

6. The most important determinant in selecting a corrective procedure for any type of ptosis is a. vertical height of the palpebrak tissue b. Age of the patient c. Amount of the levator function d. Duration of the ptosis e. Position of upper eyelid margin relative to the corneal limbus 7. A 70 year-old woman has 4 mm of right upper eyelid ptosis and 1 mm of left upper ayelid retaction. She has a heig eyelid crease in the right upper eyelid with normal levator function of both upper eyelids. Treament of the choice is a. A moderated internal tyarsoconjungtival resection (fasanella-servat operation) on the right upper eyelid b. A moderate levator recession of the left upper eyelid c. A levator aponeurosis advancement on the right upper eyelid d. A posterior approach, standart mullerectomy on the right upper eyelid

e. A frontalis muscle suspension on the right upper eyelid using a silicone rod to allow postoperative adjustment 8. A 40 year-old woman presents with a progresively enlarging clear cystic mass along the eye lid diagnosis is a. Epidermal inclusion cyst b. Apocrine hidrocytoma c. Siringoma d. Tricholofolliculoma e. Basal sel carsinoma 9. A 3 year-old girl was bitten by her pet dog. A 6-mm-wide block of upper eye-lid margin is hanging by a thread of tissue. This block of tissue left a defect in the upper eyelid approximately the same size as the tissue it self. The best do would be to a. send the block of tissue to pathology and repair the defect by approximating the two margin b. send the block of tissue to pathology and repair the defect by making a lateral canthotomyso that the skin adges ccan be approximata c. Repair the eyelid by sewing the block of tissue into its normal anatomic position d. send the tissue to pathology and repair the defect by a tranfer of the tissue from the lower eyelid e. Keep the tissue under refrigeration for later use if necessary and close the defect by approximation of the wuond edges and lateral canthotomy 10. A 74 year-old woman present with a 2-year history of the painless, progresively enlarging mass in the central aspect of the upper eyelid. This has resulted in distortion of the eyelid margin and loss of eyelashes. The most likely diagnosis is a. Sebaceous gland carcinoma b. Squamous cell carcinoma c. Amelanotic melanoma d. Basal cell carcinoma e. dermal nevus 11. Six hour after blepharoplasty, the patient complains of sudden pain near the right eye. The dressing are removed and the right eyelids are tense and ecchymotic. The first step would be to a. Open the wound to release a possible retrobulber hemorrhage b. Consider the possibility of cavernosus sinus thrombosis and corneal sensation c. measure visual acuity and check pupillary respon d. Begin treatment with ice packs e. Begin treatment with warm compresses A 75 year-old woman complains of tearing and discharge. Irrigation of lower canalliculus produces mucopurulent reflux. All the following are true except ; a. Jones testing will not reveal dye in the nose b. There is the probably a common canalicular block c. The condition is not likely to resolve with a course of antibiotics d. The most likely diagnosis is a lacrimal duct obsruction e. The treatmen is dacryocystorhinostomy

12.

13. A 14 year-old child has had tearing and discharge from the right eye since birth. Which the following statements is true? a. dye disappearance testing is likely to show no asymetry b. This condition is likely to resolve spontaneusly c. The appropriate treatment is nasolacrimal duct probing d. Punctal abnormalities are likely to be the cause e. jones I and jones II test are necessary to make the diagnosis. 14. The management of rhabdomyosarcoma of the orbit usually involves a. Lumbar puncture to rule out central nervuos system metastasis b. Exentration of the orbit c. Enucleation and orbital radiation d. Systemik chemotherapy and orbital radiation e. Radical neck disection if cervical limph nodes are involved 15. Congenital coloboma is frequently associated with : 1. Facial clefts 2. Eyelid margin inversion 3. Lacrimal deformities 4. Eyelid retraction dysgenesis 16. Acquired eyelid Disorder associated by : 1. Chalazion 2. Hordeolum 3. Eyelid edema 4. Flappy eyelid Syndrome 17. In patiens with a facial nerve paralysis, all of the following characteristics may be present except : a. eyebrow ptosisw b. Blepharopthosis c. Lower eyelid ectropion d. Epiphora e. Ocular exposure symtom 18. A patient with Telecanthus and epicanthus in inversus are correted by : 1.Y plasties 2. Z plasties 3. V plasties 4. Y-V plasties 19. Severe congenital Ectropion may give rise to : 1. Chronic epiphora 2. Blepharophimosis 3. Exposure Keratitis 4. Ichthyosis LENS & CATARACT 1. A patient with irregular astigmatism. The doctor will do corneal topography. The accuracy of corneal topography may be affected by various potential problems. Which one is correct:

1. 2. 3. 4. 2.

Tear film effects Distortion Sensitivity to focus errors Decrease accuracy of corneal power simulation measurements after refractive surgical procedure.

A 30 yo man will get corneal refractive surgery. Corneal refractive procedures can be classified as lamellar keratotomy, keratectomy, collagen shrinkage, or penetrating keratoplasty. These procedure can alter the corneal biomechanics in several ways: 1. Incisional effect 2. Tissue addition or subtraction. 3. Alloplastic material addition 4. collagen shrinkage. A patient with +6.0 D want a refractive surgery. Which one of type refractive surgery can you do for that patient: 1. LASIK 2. Intracorneal lens or ring 3. Epikeratoplasty 4. Keratotomy

3.

4. A patient with astigmatism 5. 0 D. For this astigmatism case which one of this type is correct 1. Keratotomy radial 2. Collagen shrinkage 3. Limbal relaxing incision 4. Penetrating keratoplasty 5. The patient after RK can have many complication. Potentially blinding complications occur rarely after RK. These include: 1. Perforation of the cornea 2. Bacterialkeratitis 3. Traumatic rupture of the globe occurring through a keratotomy incision 4. Vascularization of stromal scars. 6. In hexagonal keratotomy can result corneal ectasia.What is the visual problem can we find after procedure that induce corneal ectasia: 1. Glarre 2. Fluctuating vision 3. Irregular astigmatism 4. Polyopia A Patient with refractive surgery procedure. In this procedure aplus power lens is placed intra stromally to increase the curvature of the anterior cornea for the correction of hyperopia. The lens can be prepared either from donor cornea ( homoplastic or alloplastic). What kind of that procedure: a. Epikeratoplasty b. Intra stromal cornealring segment. c. Keratophakia d. Orthokeratology

7.

8.

One of many refractive surgery procedure that eliminate the complexity of the lamellar dissection and intra operative lathing of early keratomileusis procedure in which a corneal cap was dissected from the eye,shaped on a cryolathe and then repositionded with suture.What kind of this procedure: a. Keratophakia b. Epikeratoplasty c. Orthokeratology d. Intrastromal corneal ring segment Epikeratoplasty was later expanded to include the treatment of hyperopia and myopia. The following is recommendated by FDA in 1998 for selection the patient: 1. A visually disabled patient with keratoconus whois contact lenstolerant. 2. A visually disabled monocular aphake for whom secondary IOL insertion is contraindicated 3. A visually disabled binocular aphake for whom secondary IOL insertion is contraindicated 4. A patch graft used for tectonic purposes. In a refractive surgery procedure a circular rings PMMA are placed in the midperipheral corneal stroma in a lamellar channel. What is the procedure: a. Keratophakia b. Intrastromal corneal ring segment. c. Keratophakia d. Orthokeratology Intacs are approved by the FDA for low levels of myopia ( -1.0 D 3.0D spherical equivalent) but cannot correct astigmatism. Additional selection criteriafor patient : 1. With an age of 21 year or older 2. With documented stability of refraction at least 12 month prior to the preoperative examination. 3. With 1.0 D of astigmatism or less 4. In eye with a corneal keratotomy steeper than 46 D or flatter than 40 D. For the myopia or hyperopia patient can get Intacs. Intacs are not recommended for this following: 1. In patient with a low-light pupil diameter of 7.0 mm or larger. 2. In patient with systemic disease likely to affect wound healing. 3. In patient with a history of ophthalmic herpes simpleks or herpes zoster. 4. with 1.0 D of astigmatism or less. A post Intacs patient can get complication. The following is complication of Intacs: 1. Anterior chamber perforation 2. Microbialkeratitis 3. Implant expulsion 4. shallow ring segment placement A patient with corneal refractive therapy refers to the overnight use of gas permeable contact lenses to temporarily reduce myopia (-0.50 to 6.00 D of sphere with up to 1.75 D of astigmatism).This methode is:

9.

10.

11.

12.

13.

14.

a. b. c. d. 15.

Orthokeratology Keratophakia Epikeratoplasty Intrastromal corneal Ring segment.

Post operative dry eye is more common with LASIK than with PRK. How toperform preoperative evaluation: 1. Tear meniscus 2. Rose Bengal staining 3. Schirmer testing 4. History Post LASIK patient with complication come to you. He is in stage 1 diffuse lamellar keratitis. What can you find: a. Peripheral faint white blood cells, granular appearance b. Central scattered white blood cells, granular appearance c. Central dense white blood cells in visual axis. d. Permanent scarringor stromal melting A patient post LASIK with complication had flap melting, severe irregular astigmatism,corneal scarring and we should to lift that flap. What kind of the complication: a. Sterile interface inflammation b. Infectious keratitis c. Epithelial ingrowth d. Interface debris The doctor will do clear lens extraction. The following condition is suitable for this: 1. Cornea too thin 2. Cornea too flat 3. Cornea too steep 4. Refractive error exceeds the limit of excimer laser treatment. LASIK and PRK are both commonly use to treat which refractive range: a. -14 to + 8 D b. -8 to + 4 D c. -6 to +2 D d. -5 to + 5 D Which one of the following condition is an absolute contraindication to refractive surgery: a. dry eye b. glaucoma c. scleral buckling d. Herpes virus e. Amblyopia with vision < 20 / 200.

16.

17.

18.

19.

20.

CLINICAL OPTIC 1. A patient with low vision wearning +6.25+4.75 x 90 OD and +5.00+3.50 x 90 OS has an overrefraction (see the figure) of-1.00+1.25x180 OD and +0.50+1.00 x 90 OS at a distance of 1 m from a wall chart. The resultant distance refractive correction is a. +7.50+3.50 x 90 OD, +6.50+4.50 x 90 OS b. +6.50+3.50 x 90 OD, +5.50+4.50 x 90 OS c. +5.25+6.00 x 90 OD, +5.50+4.50 x 90 OS d. +5.50+3.50 x 90 OD, +4.50+4.50 x 90 OS At a distance of 1 m, the patient in Question OR1 reads the 6/9 line OD and the 6/6 line OS. The patient requests that you fill out a disability form. You record the acuity as a. 20/30 OD, 20/20 OS b. 20/180 OD, 20/120 OS c. 20/100 OD, 20/80 OS d. 20/300 OD, 20/200 OS Some patients with low vision have better reading function binocularly, wherears others read better monocularly. Which of the following is least accurate? a. When both eyes have substantially asymmetric macular function (eg, OD 20/60, OS 20/400), retinal rivalry may limit binocularity at near, and thus may affect print locailzation, cause text jump, and worsen visual blur. b. Binocular reading should be explored when the visual acuity of each eye is within two lines of each other. c. Worth four-dot testing at near is helpful in assessing peripheral fusion and binocular low-vision potential d. Asymmetric eccentric fixation enhances binocularity by providing similar extrafoveal acuity in each eye, regardless of retinal correspondence. A 26-year old man had a cataract extraction with a scleral-sutured IOL 2 years ago, after he developed a traumatic cataract with marked zonular dehiscence. Six months ago, his eye was quiet and his best-corrected visual acuity was 20/30 with -1.25 sphere. Yesterday he was hit in the pseudophakic eye. Today he complains of bulrred vision, and a slit-lamp examination shows that lens is rotated about its vertical axis. Assuming the lens is not displaced anteriorly or posteriorly, you would expect him to be. a. More myopic,with with-the-rule astigmatism b. Less myopic, with with-the-rule astigmatism c. More myopic, with against-the-rule astigmatism d. Less myopic, with against-the-rule astigmatism In prescribing telescopic devices for low vision, you must consider the optical design of the telescopic system and the tasks to be addressed. Which of the following is true ? a. The gallean telescope has a plus objctive and a minus ocular lens and produces a real and inverted image.

2.

3.

4.

5.

b. Positioning the telescope so that the exit pipil of the telescope coincides with the entrance pupil of the patients eye ensures the largestfield of view and maximum light transmission. c. Aligning the telescopes optical axis with the patients optical axis minimizes distortion. Even when the patient has an accentric visual axis of an extrafoveal preferred retinal locus. d. Alow-power minue lens used as a reading cap over the objective is positioned inside the telescopes exit pupil (between the optical center and the focal point) to reduce individual accommodative effort and permit a closer near focus. 6. Contrast sensitivity loss in the low-vision patient may adversely affect the performance of specific tasks. Which of the following statements is least accurate? a. High-freguency contrast sensitivity loss particularly aggravates near point tasks and reading performance. b. A 3x magnifier with supplemental halogen illumination can rival a. 5x magnifier using only ambient lighting. c. Telemicroscopes (eg,loupes) are often helpful when low vision is accompanied by impaired contrast sensitivity. d. Travel-related tasks and mobility issues are most affected by low-frequency contrast sensitivity loss. You examine a healthy 6-month old child with uncertain visual acuity but definitely showing a reduced visual interest. The clinical examination is unrevealing, and further testing is scheduled. Which of the following is most accurate regarding the childs visual impairment and its educational implications? a. The child should be referred immediately to the district school system so special education assessment for visual impairment can begin. b. The absence of nistagmus now and over the next 2years is an indication that special education vision sevices are probably not necessary now and will not be necessary in the future. c. Federal law mandates that the visually impaired child, beginning at age 3, be identified to the special education system. d. The physician should wait to refer a child with a possible visual impairment to the special education system until the diagnosis is established or legal blindness is confirmed. Low-vision rehabilitation is distinguished in ICD-9 coding by the level of visual impairment. Which of the following is not true regarding the 369.xx codes? a. Moderate low vision is defined as 20/70 to 20/200 acuity. b. Severe low vision is defined as 20/200 to 20/400 acuity. c. Profound low vision is defined as less than 20/400 acuity d. A specific visual field restriction, although found in the definition of legal blindness, is not a consideration in ICD-9 coding of the visual impairment. An air traffic controller with 4 D of myopia and 1 D of accommodative amplitude wants single-vision glasses for viewing a video screen 80 cm in front of her. If she uses half of her accommodative reserve, what power lenses should she be given? a. -3.00 sph b. -2.75 sph c. -3.25 sph d. -3.50 sph

7.

8.

9.

10.

A new patient wishes to be fitted with cantact lenses for an upcoming ski trip. He plans to wear the contact lenses only for recreational activities. His refractive correction is -6.00 + 0.25 x 090 OU. Which type of lenses should you recommend? a. PMMA lenses b. Spherical soft hydrogel lenses c. Rigid gas-permeable (RGP) lenses d. Toric soft lenses While performing retinoscopic on a 3-yeard-old, the retinoscopic reflex is neutralized by -3.00 sph OD and +3.50 sph OS. Assuming a working distance of 67 cm, the best prescription to give is? a. -3.00 sph OD, +3.50 sph OS b. -4.50 sph OD, plano OS c. -4.50 sph OD, +2.00 spg OS d. -3.00 sph OD, plano OS An asymptomatic 6-year-old has uncorrected snellen acuity of 20/50 OD and 20/40 OS. Ductions and versions are full, and the child is orthophoric at near and distance. Cycloplegic refraction of OD +1.00 + 4.50 x 045 and OS + 0.50 + 4.00 x135 yields no improvement in acuity. The best course of treatments is. a. Not to precsibe glasses there is no change in acuity with correction. b. To prescribe + 0.75 + 4.50 x 045 OD, and + 0.25 + 4.00 x 135 OS c. To prescribe + 0.75 +2.25 x 045 OD, and + 0.25+2.00 x 135 OS because the child might not tolerate the full amount of oblique cylinder. d. To prescribe the spherical equivalent correction because there is no change in acuity with correction. The boy in the figure on page 320 was hit in the fece with a soccer ball this morning and is complaining of binocular vertical double vision. At this last visit, he had normal vision with correction and no strabismus. The plastic surgeon wants to repair a small left orbital floor fracture that was demonstrated on a CT scan of the orbits. Before operating, the plastic sur-in each eye, and eye movements are full. If his diplopia is due solely to spectacle misalignment, you would expect that? a. The effective bese-up prism OS is causing a left hypertropia on cover testing. b. The effective base-down prism OS is causing a right hypertropia on cover testing c. The effective base-down prism OS is causing a left hypertropia on cover testing d. The effective base-up prism OS is causing a right hypertropia on cover testing.

11.

12.

13.

14. A 51-year-old electrical engineer has a refraction of -250 + 0.25 x 161 OD and -2.25 + 0.75 x 016 OS; his accommodative amplitude is 1 D. He has bifocals but complains of trouble with his intermediate vision. You write a prescription for trifocals, with an intermediate of +1.50 and a near of +3.00. He returns a week later complaining of gaps in his clear vision between a. 67 and 100 cm, 25 and 40 cm b. Infinity and 67 cm, 40 and 33 cm c. Infinity and 40 cm, 33 and 25 cm d. 67 and 100, 40 cm and 33 cm 15. A previously emmetropic and orthotropic patient is given the following glasses after bilateral cataract surgery:-0.50-2.00 x 179 OD, +0.50+1.00 x 180 OS, with + 2.50 add OU. If the

patient reads I cm below the optical centers of his lenses, the anticipated deviation in the reading position is a. a 4 right hypotropia b. a 1 right hypotropia c. a 4 right hypertropia d. a 1 right hypertropia 16. A 62 year-old retired real estate salesman consults you because of intermittent blurring and binocular diplopia while reading and driving. He often shuts his right eye to see better. He is on no medications and past medical and surgical history are unremarkable. After further questioning, the patient reports macropsia OD. Which of the following statements about spectacle correction is most appropriate? a. Iseikonic spectacles are indicated to treat his macropsia b. One prism diopter base-out prism OU (total 2) should be perscribed to help his esotropia. c. Polycarbonate lenses should be prescribed, even if refraction does not improve his acuity. d. Ultraviolet-filtering lenses should be recommended to prevent further progression of his age-related macular degeneration.

INFEKSI IMUNOLOGI 1. A 55 y.o patient presents in your office with complaints of decreased vision. On examination, you detect severe anterior basement membrane dystrophy and a cataract. Which one of the following tests would be most helpful to you to determine whether the main cause of decrease vision is the corneal pathology or the cataract? a. Potential acuity meter b. Corneal topography c. Contact lens overrefraction d. Glare testing A healthy 60 y.o man present with a 2 day history a painful rash on the right side of his forehead extending down to the eyelids. A vesicular skin lesion is also seen near the tip of his nose. Which of the folowing therapies would be most appropriate? a. Topical trifluridine 1% drops 8 times per day for 14 days b. Oral famciclovir 500 mg 2 times per day for 10 days c. Oral valaciclovir 1000 mg 3 times per day for 10 days d. Oral acyclovir 800 mg 3 times per day for 10 days A 41 y.o Japanese man with a remote history of blunt trauma in one eye but good vision and no history of ocular surgery presents with decrease vision and severe pain in both eyes. He has bilateral uveitis, alopecia, vitiligo, and recent cerebrovascular accident. There is an exudative retinal detachment in one eye. Which of the following diagnoses is the most likely? a. Sarcoidosis b. Sympathetic ophthalmia c. Behcet syndrome d. VKH syndrome A 67 y.o white female present with mild uveitis with mild vitritis and subretinal infiltrates. The condition has been minimally responsive to topical corticosteroid treatment. She has

2.

3.

4.

recently experienced weakness and confusion. Which of the following tests would be the most important to obtain at this time? a. Gallium scan b. CT-scan or MRI of the head c. PPD and chest x-ray e. Westergren sedimentation rate and C-reactive protein 5. A patient with a previous mitomycin C trabeculectomy present with a severe bleb related endophthalmitis . The VA previously was 20/20 and is now hand movements. Which of the following is not correct? a. The visual prognosis is poor b. The organism in a bleb related endophthalmitis is more likely to be H influenzae or a streptococcus species than it is in a post cataract surgery endophthalmitis c. Because the VA is better than light perception, a vitreous tap for cultures and injection of antibiotics should be performed d. Endophthalmitis may present months or years after glaucoma filtering surgery A 35 y.o Japanese women present with slate gray pigmentation of the episclera of the right eye with association pigmentation of the periocular skin. Which of the following statement applies to her? a. Her lifetime risk of uveal melanoma is about 1 in 400, significantly greater than the risk of the general population b. Malignant melanoma can develop in the skin, uvea, or orbit but not in the conjunctiva c. The risk of malignant transformation is the same in all patients regardless of the their complexion d. This condition is always unilateral A 33 yoman was involved in a motor vehicle accident that resulted in a broken windshield. Slit-lamp examination reveals multiple glass foreign bodies embedded at various levels of the stoma. All wounds were seidel negative. Which of the following is the most appropriate treatment strategy? a. The patient should be taken to the operating room and all superficial and deep fragments should be removed immediately b. Ointment should be place in the eyes since superficial and deep fragment will eventually be extruded with the time c. Eyes should be patched with the follow up the next day d. All exposed glass fragment should be removed and deeper fragments left in place Which of the following can be related to spontaneous hyphema? a. Herpetic disease b. Retinoblatoma c. Juvenile xanthogranuloma d. All of the above Currenty the most prevalent refractive surgery is : a. Myopic LASIK b. Hyperopic LASIK c. Cataract removal with IOL implantation

6.

7.

8.

9.

d. LASEK 10. All of the following organisms can invade an intact corneal epithelium except : a. Pseudomonas aeruginosa b. N meningitidis c. C diphtheriae d. Shigella A hospitalized patient on IV antibiotics for pneumonia develops floaters and blurred vision in one eye. Funduscopic evaluation reveals multiple yellow white choroidal lesion with indistinct margin and vitreous fluff balls. Which of the following presents the most appropriate management? a. Inquire whether the patient has had recent exposure to cats or uncooked meat b. Culture the patients blood and catheter tips on multiple media and begin IV antifungal medication c. Supplement the IV antibiotics with topical and subconjunctival antibiotics to increase ocular penetration d. Prescribe systemic corticosteroid to suppress the vitritis A 64 y.o woman who had undergone cataract surgery 6 months earlier develops an anterior chamber reaction and low grade anterior vitreous inflammation following Nd:YAG laser capsulotomy of a white plaquelike opacity of the posterior capsule. The uveitis initially respon to topical steroids but recurs as the steroids are withdrawn.The most likely diagnosis is a. Phacoantigenic endophthalmitis b. Sympathetic ophthalmia c. Uveitis-glaucoma-hyphema syndrome d. Propionibacterium acnes endophthalmitis Which of the following patients would benefit most from steroid therapy? a. A 9 y.o girl with JRA,moderate anterior chamber flare, but no cell b. A 22 y.o woman with pars planitis,anterior vitreal cells,but no CME c. A 44 y.o woman with Fuch heterochromic iridocyclitis, fine KP, and anterior chamber cell d. A 50 y.o man with herpes simplex keratouveitis, disciform corneal edema, keratic precipitate, and chamber flare cell A 64 y.o man developsbilateral decreased vision and floaters. Examination reveals clumps vitreous cells; subretinal infiltrates; and a white quiet eye.The vitritis is not responsive to steroid. Themost likely associated finding would be a. Tumor cells in the CSF b. Positive VDRL serology c. Hilar Lymphadenopathy on chest x-ray d. Pneumocystis carini in sputum specimens Peripheral retinal granuloma can be found in all of the following except : a. Ascariasis b. Toxoplasmosis c. Sarcoidosis d. Syphilis

11.

12.

13.

14.

15.

16.

Diagnostic criteria supporting a diagnosis of Behcet syndrome include all of the following except : a. Genital ulcers b. Oral ulcers c. Keratoderma blennorrhagicum d. Erythema nodosum A 9 y.o boy with a history of atophy present with a seasonally recurrent bilateral conjunctivitis and complains of blurred vision for 1 week. Giant papillae are seen upon lid eversion. All of the following could also seen on the slit lamp exam except a. Vascullar pannus and punctate epithelial erosion involving the superior cornea b. An oval epithelial ulceration with underlying stromal opacification in the central cornea c. Limbal follicles d. Conjunctival symblephara A 30 y.o woman with chronic irritation in both eyes is found to have mild papillary conjunctivitis, punctuate staining of the superior cornea and conjunctiva of both eyes and filaments attached to the upper limbus of one eye. Bulbar conjunctival biopsy of the involved area showed keratinization, acanthosis, intracellular glycogen granules. Which of the following diagnostic tests would be most appropriate? a. Thyroid functions testing b. Serum RF c. HLA typing d. Histamine level of the tear film Steven Johnson Syndrome a. Is also referred to as erythema multiforme minor b. Is generally a disease of elderly patients c. Is commonly associated with drug hypersensitivity d. Rarely presents as an acute febrile episode A 59 y.o woman presents with the recent of a flat, darkly pigmented macule of the left lower palpebral conjunctiva. Of the following conditions, which would be most likely? a. Blue nevus b. Primary acquired melanosis c. Benign acquired melanosis d. Lentigo senilis

17.

18.

19.

20.

CLINICAL OPTIC 1. A patient comes for refractive surgery with keratometry readings of 43,0 D/ 42,0 D and a manifest refraction of -9,5 D. If LASIX were performed,you would expect the postoperative average keratometry reading to be : (102) a. 36,8 D b. 36,3 D c. 37,3 D d. 34,0 D 2. The clinician should be familiar with levels ofillumination frequently recommended by engineers. Recommended illumination levels are (in foot candles) : (16-17) 1. Office : 150 2. Kitchen :70 3. Projector : 10 4. Operating table : 250

3. Retinoscopy is usually performed using the plano mirror setting. Characteristics of the retinoscopy reflex are : (128) 1. speed 2. width 3. brilliance 4. colour 4. A six years old boys with hyperopia unless there is esodeviation or evidence of reduced vision,our management is: (146-147) a. It is not necessary to correct hyperopia b. full correction according to cycloplegic retinoscopy c Minimum correction according to cycloplegic retinoscopy d. Correction is waiting for until 8 years old e. Correction without cycloplegic 5. Magnification of approximately 20-30%, altered depth perception resulting from the magnification, pincushion distortion, ring scotoma generated by prismatic effects at the edge of the lens, & jack-in-the-box are some problems of correcting : (164) a. contact lens b. RGP c. Prebylasix d. Aphakia e high myopia 6. Monocular Diplopia is frequent and confusing complaint in general ophthalmic practice, causes by : (170-171) 1. Irregular astigmatism 2.decentered contac lens 3. Double reflection inspectacle lenses 4. Post strabismus surgery 7. Key optical considerations resulting from contact lens use are : (177) 1. Field of vision 2. Image size 3. Accomodation 4.tear lens 8. Conditions and circumstances where bandage contact lenses might be useful include : (202) 1. Bells palsy 2.Reccurent erotions 3. Bullous keratopathy 4.Keratitis 9. Sometimes an IOL maybe inserted in an eye that already has an IOL, usually to correct a postoperative refractive error. In this case, theoretically, there is no change in any of the optical parameters of the eye,so calculation of the needed second IOL power is straightforward geometrical optics : (224-229) a.Multifocal IOL b.Piggyback IOL c. Standard IOL d. Diffractive multifocal IOL e.Accomodating IOL 10. A small change of vertex distance produces considerable blurring of vision,especially for patients with : (144) 1. An isometropia 2. High myopia 3. Silicon IOL 4. Aphakia

SOAL SOAL BOARD UNAIR JAN 08


1. A 20 years old woman came with redness on her both eyes, she also suffer fom rhinitis allergic and asthma. Which one is the most signified for this symptom: a.Foreign body sensation b.Itching c.Sandy feeling d.Deep pain e.Lid pain Answer : B (AAO 2006-2007 section 8 Externa disease and cornea : pg. 208)

2. Patiens came with red nodular mass, location surrounding the eye lashes, she complains of tenderness and painful. The diagnosis is : a. Internal hordeola b.Styes c.Chalazion d.Periorbital celulitis e.Meibomian glan dysfunction Answer : B (AAO 2006-2007 section 8 Externa disease and cornea : pg. 156) 3. The management of the disease above is : a.Culture b.Topical antibiotic c.Systemic antibiotic d.Warm compress e.All above is true Answer : D (AAO 2006-2007 section 8 Externa disease and cornea : pg 157) 4. A 6 years old girl present with diffuse, erythematous & edema of the eye lids. Lids are tenderness to touch and swollen, visual acuity and ocular motility are normal. The most common organism can cause this problem is: a.Streptococcus epidermidis b.Staphylococcus aureus c.Klebsiella sp. d.Neiseria meningitidis e.Pseudomonas aeruginosa Answer : B (AAO 2006-2007 section 7 orbit eyelid, lacrimal system : 41-42) A man, 35 years old present with serous discharge and red eye. Which of the folowing statement aplies to him: 1. pattern of redness tends to be concentrated in the palpebra are rather than near the cornea 2. palpable lymph node auricular 3. discharge swab revealed lymphosite dominant 4. folicular conjugtivitis Answer :E (AAO 2006-2007 section 8 Externa disease and cornea : pg 24-25)

5.

6.

A 65 years old lady who consumes many drugs for her diseases complained burning, gritty sensation, reflex tearing and mild redness. Which of these drugs can influence her symptoms? 1. systemic antihistamine 2. diuretic 3. antidepresant 4. non steroid inflamatory drug Answer :B (AAO 2006-2007 section 8 Externa disease and cornea: pg 81) 7. We performed several examination for her, and revealed : 1. tear meniscus < 0,1 mm 2. break up time more rapid 3. schirmer test < 5 mm 4. fine and granular epithelial keratopathy was shown by Rose Bengal Answer : E (AAO 2006-2007 section 8 Externa disease and cornea: pg 73)

8.

A Female model, 17 years old with cosmetic contact lens wear present with redness, tearing , pain, photophobia and flourescein test positive at 3 and 9 oclock in the cornea. The management were: a. prescription of topical anesthesia b. cyclopentolate 1% topical c. anti inflamation eye drop 3 tmes daily d. antibiotic ointment Answer : C (AAO 2006-2007 section 8 Externa disease and cornea: pg 406-407) 9. Corneal dystrophy of Bowmans layer is a. Previously known as Reiss Buckler dystrophy and Thiel Behnke dystrophy b. Progresive autosomal dominan c. Thiel Behnke dystrophy demonstrated curly fibers on electron microscopy and is linked tochromosom 10q24 d. Appears at third decade of life Answer : A (AAO 2006-2007 section 8 Externa disease and cornea : 315)

10. Macular dystrophy is a. autosomal recessive inheritance b. amyloid deposit concentrated in the anterior stroma c. gene identified on chromosom 16 q22 d. affected the central corneal and do not involve cornea endothelium Answer : B (AAO 2006-2007 section 8 Externa disease and cornea : 310) 11. Which one of these component imunology in healthy conjungtiva is absent: a. neutrophil b. lymphosite c. eosinophyl d. macrophage e. plasma cell Answer C (AAO 2006-2007 section 8 Externa disease and cornea : 195) 12. A man of 20 years old, came to the hospital with complain right red eye, tearing, eyelids edema with normal visual acuity . A few red eye disorders can threaten vision,except: a.Corneal infection b.Scleritis c.Iritis d.Blepharoconjungtivitis e.Orbital celulitis Answer : D (AAO: Eye care skills on CD ROM 2001 slide 1.) 13. The corneas imuno previlege is due to multitude of factors, including : 1. absence of blood vessels 2. absence of lymphatics 3. expression of neuropeptide such as alpha-Melanine Stimulating Hormone (alpha MSH) in the cornea 4. absence of expression Transforming Growth Factor Answer A (AAO 2006-2007 section 8 Externa disease and cornea : 196) 14. Graft versus Host disease is a relative common complication of allogeinic bone marrow transplantation. In this condition, the grafted cell attack the patients tissues including the eyes. Which tissues the eye can affected? a. Palpebra b. Conjungtiva c. Uvea d. Lacrimal gland Answer C (AAO 2006-2007 section 8 Externa disease and cornea : 223-224) 15. Some disorders give symptom of photophobia that indicates problems arising from anterior segment . Which of the following condition can cause photophobia?: a. corneal abrasion

b. iritis c. acute glaucoma d. corneal edema Answer : A (AAO: Eye care skills on CD ROM 2001 slide 5) 16. Interstitial keratitis is nonsupurative inflamation of the corneal stroma that features cellular infiltration and usually vascularization without primary involvement of the epithelium or endothelium. The most common cause to be linked with interstitial keratitis is : a. M tuberculosa b. Rubeola (measles) c. Clamydia trachomatis d. Onchocerciasis e. Syphilis Answer ; E (AAO 2006-2007 section 8 Externa disease and cornea : 226) 17. A 51 years old lady present with a chronic, progressive, painful, idiopathic ulceration in peripheral corneal and epithelium. The ulcer starts at periphery of the cornea and spreads circumferentially and then centripetally, with leading undermined edge of de-epithelized tissue. This lady suffered from : a. idiopathic peripheral ulcerative keratitis b. Superior Limbus Keratitis c. Ulcus mooren d . Ulcus marginalis e. Keratoconjungtivitis Sicca Answer : C ( AAO 2006-2007 section 8 Externa disease and cornea :232) 18. Which one following examination is not present in this disease: a. Deficiency of Supressor T cell b. Increased level of Ig A c. Increased level of Ig G d. Increased concentration of plasma cells and lymphocytes in the conjungtiva adjacent to the ulcerated areas e. Tissue-fixed imunoglobulins and complements in the conjungtival epithelium and peripheral cornea Answer : C (AAO 2006-2007 section 8 Externa disease and cornea : 232) 19. The conjungtival biopsy can be helpful in evaluating chronic conjungtivitis and unusual cular surface diseases. Which one of the following condition unnecessary to do conjungtival biopsy : a. Superior limbic Keratitis b. Pterygium c. Conjungtival lymphoid tumors d. Graft versus Host disease e. Cicatrical pemphigoid Answer : B (AAO 2006-2007 section 8 Externa disease and cornea: 427) 20. Seorang pemuda berusia 18 tahun datang dengan riwayat kecelakaan lalu lintas 1 jam sebelum ke Rawat Darurat. Dari pemeriksaan didapatkan laserasi pada kornea mata kanan. Hasil pemeriksaan dibawah ini dapat memastikan adanya perforasi bola mata yaitu a. Bilik mata depan dangkal b. Kemosis konjungtiva c. Hipotoni d . Seidel tes positif e. Perdarahan subkonjungtiva Answer : D (AAO 2006-2007 section 8 Externa disease and cornea: 409) 21. The principal photodisruption laser used in clinical ophthalmology is : a. b. c. Cardondioxide Carbon Nd: YAG laser

d. e.

Argon Hollonium Jawab : C

22. An Object positioned inside the focal point of a plus spherical lens will produce image that a. b. c. d. e. Minified, inverted, real Minified, upright, real Magnified, inverted,virtual Magnified,upright,virtual Magnified,upright, real

Jawab : D

23. Starting point of reading add for low vision patient with 6/24 best corrected vision is .. a. b. c. d. e. +3.00 D +3.50 D +4.00 D +5.00 D +6.00 D

Jawab : C 24. A modified A-constan of an IOL for patient with axial length of 23,5 cm by using SRK II formula is a. b. c. d. e. A+3 A+2 A+1 A-A A-0,5

Jawab : D 25. Equivalent of two spherocylindrical lens +1.00 -2.50 x90 and +1.00 +2.00 x180 a. b. c. +2.00 -0.50 x 90 -0.50 -4.50 x 180 +1.50 +0.50 x 180

d. e.

+1.00 -4.50 x 90 +0.50 +2.00 x18 Jawab : B

26. Streak retinoscopy of working distance of 50 cm . the horizontal axis is neutralized first with a spherical lens -4.00 D and then the vertical axis is neutralized with cylindrical lens +2.00 D what is the refraction error a. b. c. d. e. -4.00 -2.00 x 180 -1.00 +2.00 x 90 -6.00 +2.00 X90 +4.00 -2.00 x 90 -6.00 +1.00 x180

Jawab : A

Question no 7-12 Object is placed in front of +3.00 D lens. The lens +3.00 D in turn is position 50 cm in front of a +2.00 D lens.
27. Wher does the 3 D lens from intermediate image ? a. b. c. d. e. At optical infinity In front of the lens 0,5 m in front of the lens 2 m in front of the lens 1 m in front of the lens

Jawab : A 28. Describe the intermediate image a. b. c. d. e. Inverted, real, magnified inverted, real, minified inverted, virtual, magnified inverted, virtual, minified upright, virtual, minified

jawab : C 29. what is the size of the intermediate image as compare to the object? a. b. c. d. e. Indeterminate On fourth the suze Half size Same size Twice size

Jawab : E 30. what is the location of the final image ? a. b. c. d. e. 1 m in front of the second lens 0,5 m in front of the second lens 0,25 in front of the second lens 1 m behind the second lens At optical infinity

Jawab : C 31. Seorang wanita, 56 tahun mengalami ptosis kelopak mata kanan sebesar 4 mm dan 1 mm retraksi kelopak mata kiri atas. Dia mempunyai lid crease yang tinggi dan fungsi levatornya baik pada kedua mata. Pilihan terapinya adalah : a. b. c. d. e. Reseksi tarsoconjungtiva interna (Fasanella Servat operation) pada kelopak atas mata kanan Resesi levator kelopak atas mata kiri Levator aponeurosis advancement pada kelopak atas mata kanan Pendekatan posterior, menggunakan mullerectomy standad pada kelopak atas mata kanan Suspensi otot frontalis pada kelopak atas mata kanan menggunakan silikon agar dapat dinilai kembali paska operasi Jawab : C (AAO 2006-2007 section 7 Orbit, Eyelid, lacrimal system hal 220-223) 32. Seorang laki-laki usia 25 tahun mendapatkan trauma tajam pisau yang mengenai kelopak atas mata kanannya 1 jam sebelum ke rumah sakit. Pernderita sadar dengan GCS 4-5-6, dari pemeriksaan didapatkan laserasi sepanjang 15 mm dan 12 mm terletak diatas garis bulu mata. Terdapat ptosis 7 mm pad mata kanan dan sedikit bengkak. Setelah dilakukan evaluasi dengan tepat pada bola mata, terapi terbaik yang diberikan adalah: a. b. c. Jahit robekan kulit dan tunggu beberapa bulan agar ptosisnya membaik Jaga luka tetap bersih dan tunggu bengkaknya berkurang dalam 2-3 hari untuk dilakukan tindakan Jahit lapisan otot dan lapisan kulit secara terpisah

d.

Eksplorasi luka untuk memeriksa aponeurosis levator dan usahakan untuk menempelkan kembali jika mengenai tarsus setelah itu lapisan otot dan kulit dijahit. Jahit laserasi kulit dan lekatkan kelopak mata ke alis untuk beberapa hari agar mempercepat penyembuhan Jawab : D (AAO 2006-2007 section 7 Orbit, Eyelid, lacrimal system hal

e.

33. The most important determinant in selecting a corrective procedure for any type of ptosis is : a. b. c. d. e. Vertical height of the palpebral fissure Age of patient Amount of levator function Duration of ptosis Position of the upper eyelid margin relative to corneal limbus Jawab : C (AAO 2006-2007 section 7 Orbit, Eyelid, lacrimal system hal 221) 34. Pertanyaan di bawah ini yang benar mengenai obstruksi Duktus Nasolakrimal kongenital : 1. 2. 3. 4. Sebagian besar obstruksi terbuka spontan 4 6 minggu setelah lahir Segera dilakukan probing Biasanya disebabkan gangguan pada katub Hassner Turbinate infracture dilakukan rutin pada bayi usia 3 bulan Jawab : B (AAO 2006-2007 section 7 Orbit, Eyelid, lacrimal system hal 261-263) 35. Seorang wanita usia 70 tahun datang dengan keluhan kedua matanya sering berair. Pada pemeriksaan didapatkan entropion, trichiasis dan dilakukan tes snap back dengan hasil positif. Beberapa tanda klinis yang menunjukkan disinsersi pada retraktor adalah: 1. 2. 3. 4. ptosis palpebra inferrior forniks inferior lebih dalan daripada normal terdapat sedikit atau tidak ada pergerakan kelopak mata bawah pada saat penderita melihat ke bawah terdapat retraksi otot orbikularis Jawab : A (AAO 2006-2007 section 7 Orbit, Eyelid, lacrimal system hal 202)

36. Ligamentum transversa superior adalah : a. Ligamentum Lockwood b. Ligamentun Soomering c. The ROOF

d. Ligamentum Whitnalls Jawab : D (AAO 2006-2007 section 7 Orbit, Eyelid, lacrimal system hal 142) 37. Seorang laki-laki 60 tahun datang dengan basalioma pada kelopak bawah mata kiri, dilakukan eksisi luas yang meninggalkan defek lebar (80% kelopak bawah mata kiri)> metode yang dapat digunakan untuk merekonstruksi defek adalah : 1. 2. 3. 4. Semicircular flap Modified hughes procedure Cuttler beard flap Mustarde flap

Jawab : D (AAO 2006-2007 section 7 Orbit, Eyelid, lacrimal system hal 192) 38. Socket Anoftalmia yang dapat diterima secara fungsional dan kosmetika seharusnya adalah : 1. 2. 3. 4. Protesa nyaman dan terlihat menyerupai mata normal Palpebra tampak normal dan tonus adekuat untuk mensuport protesa volume implant orbita mencukupi di tengah orbita orbita Pergerakan transmisi implan baik Jawab : E (AAO 2006-2007 section 7 Orbit, Eyelid, lacrimal system 121) 39. Dalam menangai laserasi full thickness kelopak mata atas akibat trauma yang mengenai tepi kelopak mata , seorang oftamologis sebaiknya mengerti struktur anatomi kelopak mata yang normal. Berdasarkan urutannya : a. Kulit, preaponeurotic fat, septum, otot orbicularis, aponeurosis levator, otot Mullers, conjungtiva b. Kulit, otot orbicularis, preaponeurotic fat, septum, otot Mullers, aponeurosis levator, conjungtiva c. d. Kulit, preaponeurotic fat, otot orbicularis, septum, aponeurosis levator, otot Mullers, conjungtiva Kulit, otot orbicularis, septum, preaponeurotic fat, aponeurosis levator, otot Mullers, conjungtiva Jawab : D (AAO 2006-2007 section 7 Orbit, Eyelid, lacrimal system hal 139-140) 40. Epicantus is medial canthal fold that may result from immature midfacial bones or a fold of skin and subcutaneus tissue. The type of epicantus if the fold is most prominent in the lower eyelid : a. b. c. d. Epicantus tarsalis Epicantus palpebralis Epicantus supraciliaris Epicantus inversus

Jawab : D (AAO 2006-2007 section 7 Orbit, Eyelid, lacrimal system : 156)

Soal lain tentang :

1. 2. 3. 4. 5. 6. 7.

Duane syndrome Intermitten exotropia Refractive accomodative esotropia Monofixation syndrome 6th nerve palsy Essential infantile esotropia Operasi exotropia tanpa overaksi otot oblique

40 30 30 20 30

8. 9.

Visual confusion Mobious syndrome

10. Primary action inferior oblique 11. Corneal neovascular pada contact lensa (hal 206) 12. Terapi Bandage Contact lens (hal 202) 13. Keuntungan RGP (hal 191) 14. Ortho keratology (203)

KOI Board Examination Questions from Andalas University Padang, West Sumatra

1. A healthy 26 years old man came to Ophthalmology clinic with sudden blurred vision, micropsia, and decreased color vision. Hes already had the same condition before. The patient was fired from his job one month ago. Which is the treatment for the patient: A. Observation and bed rest. B. Oral and Systemic Corticosteroids. C. Topical Midriatic. D. Carbonic Anhidrase Inhibitor. E. Neurotrophic vitamins. (Answer: D) (AAO 2005 Section 12, pg 51) 2. A 20 years old man came to an Ophthalmology clinic in St. Louis, Ohio. He complained of visual loss, metamorphosia, and paracentral scotomata since he started working in a chicken farming. Other probable signs of his ophthalmic signs, except: A. Punched-out chorioretinal lesions. B. Juxtapapillary atropic pigmentary changes. C. Vitritis. D. Choroidal neovascularization E. Could not be determined from given information (Answer: C) (AAO 2005 Section 12, pg 79) 3. A 56 years old uncontrolled diabetic woman with history of diabetes for 15 years, complained of severe blurred vision since the last three years. She had history of hospitalization twice and insulin-dependent before meal. Her fundus examination shown moderate neovascularization elsewhere with vitreous hemorrhage. The best treatment as DRS recommendation because of its highest risk of Severe Visual Lost, for the patient is A. Pan Retinal photocoagulation treatment. B. Early surgical vitrectomy. C. Fundus examination in the next 6 months. D. Control of blood glucose. E. Control of kidney functions as soon as possible. (Answer: A) (AAO 2005 Section 12, pg 109) 4. A 39 years old woman was referred from Department of Internal Medicine to the Department of Ophthalmology with uncontrolled blood glucose level. Her fundus examination showed PDR. This could be classified as High-risk PDR because of: 1. Shes in premenopause age 2. Presence of moderate neovascularization with vitreous hemorrhage 3. She has vitreous hemorrhage after severe blunt ocular trauma 4. Presence of mild neovascularization of the disc with vitreous hemorrhage. (Answer: C 2 & 4) (AAO 2005 Section 12, pg 109) 5. A 9 years old boy came with complains of intermittent itching on both eyes that diminished if she was in cold weather condition, copious mucous discharge. His examination shown diffuse papillary hypertrophy, serpiginous corneal ulcer. The patient is should be treated with: 1. Tear film artificial 2. Topical corticosteroid 3. Mast cell stabilizer 4. Cyclosporine (Answer: B 1 & 3) (AAO 2005 Section 8, pg 197)

6. A 48 years old woman with history of recurrent red eye on her right eye, tearing, pain, photophobia and phthisis bulbi on the left eye. Her examination shows ulcer in right corneal periphery. Her laboratory finding revealed parasitemia. The effective treatment for this patient is A. Contact lens + oral corticosteroid B. Topical corticosteroid + contact lens C. Conjunctival flap + topical corticosteroid D. Amnion graft + conjunctival flap + topical cyclosporine E. Amnion graft +bare sclera +topical cyclosporine (Answer: E) (AAO 2005 section 8, pg 221) 7. A 40 years old woman came with complains of blurred, red, and pain in right eye, with history of cataract surgery with IOL implantation in 10 days ago. Preoperative condition that had high risk of triggering this condition, are: 1. Severe chronic blepharitis. 2. Lachrymal drainage abnormalities 3. Cicatrical conjunctivitis 4. Corneal cicatrix (Answer: A 1, 2, & 3) (AAO 2005 Section 8, pg 214) 8. A 12 years old boy complains of proptosed right eye, without pain or blur. His ophthalmic examination shows axial proptosis, VA 20/20 ODS. CT scan revealed isodent intraconal mass. Recommended treatment for this patient: A. Observation only B. Surgical excision C. Biopsy in order to determine etiology D. Chemotherapy E. Radiation therapy. (Answer: A) (AAO 2005 Section 7, pg 74) 9. A 25 years old man with history of post cranial trauma complains of blurr eye. His CT Scan examination revealed fracture of orbital floor. Treatment for this patient are: 1. Oral steroids 2. Systemic antibiotics 3. Observation for 7-10 days 4. Surgical (Answer: B 1 & 3) (AAO 2005 Section 7, pg 104) 10. Surgical indication for above case (question number 9) are: 1. Enophthalmus exceeding 2 mm 2. Large fracture involving at least half of the orbital floor 3. Diplopia with limitation of up gaze and/or down gaze within 30o of the primary position 4. Orbital hemorrhage (Answer: A 1, 2 & 3) (AAO 2005 Section 7, pg104) 11. A 47 years old man with scar on his left forehead. In the ophthalmology examination, VA 5/15, fine keratic precipitate, flare and cell was negative in anterior chamber, corneal sensibility was decreased.

The most probable diagnosis for this patient is: A. Keratitis HZO B. Keratitis HSV C. Keratouveitis HZO. D. Keratouveitis HZV. E. Viral Anterior Uveitis. (Answer: C) (AAO 2005 Section 8, pg 147) 12. A 5 years old girl was consulted from Pediatric Department with intermittent red eye and joint sickness. There also lymphadenopathy and splenomegaly. In eye examination was found blurred vision, lens opacity, red eye, corneal edema RE, band keratopathy, and anterior uveitis in both eyes. X-ray examination revealed bamboos appearance. In the patient, management that not be recommended: A. Topical and systemic corticosteroids B. Topical and systemic antiglaucomas C. Topical and systemic NSAIDs D. Cataract extraction and IOL implantation in RE E. Scrapping or chelation with sodium EDTA in both eyes (Answer: D) (AAO 2005 Section 9, pg 143) 13. A 20 years old man was consulted from Internal Medicine Department with redness in his right eye. His hemoglobin was 3 mg %, CD4 12, Ig G antitoxoplasma was increased, thoracic X-ray of TB duplex. His fundus examination shown massive hemorrhage. The diagnosis of the patient is A. CMV retinitis B. Posterior uveitis C. Anemia. D. Toxoplasma E. CRVO. (Answer: C) (AAO 2005 Section 9, pg 242) 14. A 39 married woman came with red and painful on the right eye since 1 week ago. In eye examination: VA light perception, red eye, palpebral margin edema, anterior chamber: flare (+), membrane(+), occlusion pupil, lens not clear, USG shows vitreous opacity, history of trauma was unclear. The patient complained mild deafness. Laboratory finding: Mantoux test 14 mm, X ray examination revealed TB duplex. The most probable diagnosis of the patient is: A. Anterior uveitis B. Endophthalmitis C. Behcet syndrome D. Vogh Koyanagi Harada syndrome E. Sarcoidosis (Answer: B) (AAO 2005 Section 9, pg 207) 15. A 19 years old man, whos a contact lens wearer for 4 months only on the right eye, came to Ophthalmology clinic, complained itching, pain, decreased visual acuity. In eye examination shown red eye, mucous tear film, papil hypertrophy of tarsal conjunctiva. The most probable diagnosis of the patient was based on the: A. Hypersensitivity type 1 B. Hypersensitivity type 2 C. Hypersensitivity type 3

D. Hypersensitivity type 4 E. Hypersensitivity type 5 (Answer: A) (AAO 2005 Section 8, pg 186) 16. A 70 years old man came with complain of blurring of RE, headache with nausea and vomiting. Eye examination shown IOP 62 mmHg, hyperemic conjunctiva, corneal edema, narrow anterior chamber. LE was normal, with narrow anterior chamber. Most common mechanism of etiology: A. Lens luxation. B. Pupillary block. C. PAS. D. Aqueous misdirection E. Anteriorly iris caused by tumor (Answer: B) (AAO 2004 Section 10, pg 101-102) 17. Medical treatment that should not be given to the patient (question number 16) is: A. Pilocarpine 1%-2% B. Timolol maleate C. Carbonic anhidrase inhibitor D. Manitol 20 % E. Apraclonidine (Answer: E) (AAO 2004 Section 10, pg 101-102) 18. To determine whether patients trabecular angle was synechia or apposition (question number 16), its better to examine him with the type of gonioscopy: A. Goldman type goniolens B. Zeiss type goniolens C. Koeppe type goniolens D. Barkan type goniolens E. Wurst type goniolens (Answer: B) (AAO 2004 Section, pg 72)

19. A 24 years old man came with blurring and pain of RE. Eye examination shown IOP 53 mmHg, hyperemic conjunctiva, corneal edema, flare (+) in anterior chamber. Gonioscopy revealed normal angle with khaki colored cell in inferior trabeculum. Two months ago, patient had ocular trauma that causing vitreous hemorrhage. The right diagnosis of the patient is: A. Ghost cell glaucoma. B. Hemolytic glaucoma. C. Angle recess glaucoma D. Lens induced glaucoma E. Glaucomatocyclitic crisis (Answer: C) (AAO 2004, Section, pg 95-98)

20. A 55 years old woman had benign tumor operation of inferior palpebral margin, mass dimensions was > 50 %. In the palpebral reconstruction, anterior lamellar can be taken from pre auricular or retro auricular skin. This method is known as: A. Modified Hughes procedure B. Mustard procedure C. Tenzel Flap procedure D. Semicircular Pedicle Flap E. Cuttler-Beard Procedure (Answer: A) (AAO 2005 Section 7, pg 192) 21. A 25 years old woman came to Ophthalmology clinic with complain of proptosed of LE since 1 month ago. LE became more proptosed since she was pregnant. CT Scan shown encapsulated homogenous mass. Recommended treatment for the patient was excisional. The right diagnosis for the patient: A. Hemangioma cavernous B. Hemangiopericytoma C. Lymphangioma D. Arteriovenous fistula E. Orbital varices (Answer : A) (AAO 2005 Section 7, pg 67)

SOAL SOAL unhas 2008 Seorang pasien umur 71 tahun datang ke poliklinik mata dengan keluhan visus menurun secara perlahan-lahan yang disertai adanya gangguan penglihatan sentral. Segmen anterior bola mata relatif normal. Pada pemeriksaan funduskopi didapatkan adanya lesi berbentuk bulatan kecil-kecil berwarna kekuningan pada level RPE di area makula. 1.Diagnosis yang paling tepat pada kasus diatas adalah : a.Non neovascular AMD b.Neovascular AMD c.CME d.Pattern dystrophy of RPE e.CSCR Jawaban : A (AAO section 12, hal 55-87)

2.Pemeriksaan pendukung yang paling utama yang seharusnya dilakukan pada pasien tersebut (skenario 1) diatas adalah : a.FFA b.ERG c.Color blindness test d.ICG e.OCT Jawaban : A (AAO section 12, hal 55-87) Seorang wanita umur 47 tahun datang ke poliklinik mata dengan keluhan gangguan penglihatan terutama pada senja hari. Bapak dan saudara laki-lakinya juga menderita hal yang sama. Walalupun segmen anterior bola mata tidak ditemukan kelainan akan tetapi terlihat gambaran pigmen-pigmen hitam tersebut di empat kuadran mid perifer retina. 3.Pemeriksaan pendukung untuk penegakan diagnosis yang tepat pada kasus diatas adalah: a.ERG dan visual field testing b.ERG dan FFA c.Visual field testing dan FFA d.ERG dan Ishihara color testing e.FFA dan Ishihara color testing Jawaban : A (AAO section 12, hal 203-212) 4. Buta senja (nictalopia) yang dirasakan oleh pasien tersebut diatas oleh karena adanya kerusakan fungsi retina pada level : a.Sel-sel RPE b.Sel-sel photoreseptor c.Sel-sel ganglion d.Lapisan serabut syaraf e.Sel-sel cone di area makula Jawaban : B (AAO Section 12, hal 207) 5. A 40-year-old woman present with a progressively enlarging clear cystic mass along the

eyelid margin.The most likely diagnosis is a.Epidermal inclusion cyst b.Apocrine hidrocystoma c.Syringoma d.Trichofolliculoma e.Basal cell carcinoma Jawaban : B 6. Seorang perempuan 74 tahun dengan riwayat nyeri, pembesaran massa yang progressif di bagian sentral palpebra superior selama 2 tahun. Didapatkan distorsi pada margo palpebra dan madarosis. Diagnosis yang paling tepat adalah a.Glandula sebaceus carcinoma b.Squamos sell carcinoma c.Amelanotic melanoma d.Basal cell carcinoma e.Dermal nevus Jawaban : D 7. Enam jam setelah dilakukan blepharoplasty bilateral, pasien mengeluh tiba-tiba nyeri pada mata kanan. Setelah jahitan dilepas, tampak palpebra bengkak dan ekimosis. Langkah pertama yang tepat adalah : a.Membuka luka untuk mengelimansi kemungkinan retrobulbar hemorrhagic b.Memikirkan kemungkinan trombosis sinus cavernosis dan uji sensitifitas kornea c.Mengukur visus dan uji respon pupil d.Mulai pengobatan dengan kompres es e.Mulai pengobatan dengan kompres hangat Jawaban : C 8. Anak 14 tahun dengan keluhan lakrimasi dan sekretd ari mata kanan, dialami sejak lahir. Dimana pernyataan yang paling tepat di bawah ini : a.Dye disappearance testing menunjukkan tidak adanya asimetris b.Kondisi ini dapat sembuh spontan c.Terapi yang diutamakan berupa probing ductus nasolakrimal d.Abnormal pungtum bisa menjadi penyebab

e.Perlu dilakukan Jones 1 dan 2 untuk menegakkan diagnosis Jawaban : C 9. Seorang laki-laki 30 tahun, dengan trauma terkena pisau pada palpebra superior mata kanan, dialami 1 jam MRS, pasien sadar dan ditemukan laserasi sepanjang 15mm dengan lokasi kira-kira 12 mm diatas silia. Pasien mengalami ptosis 7 mm di bagian kanan dan edema minimal. Sesudah dilakukan pemeriksaan bola mata,terapi yang terbaik adalah a.Perbaikan laserasi kulit dan menunggu beberapa bulan untuk perbaikan ptosis b.Menjaga kebersihan luka dan menunggu proses inflamasi mereda dalam 2-3 hari c.Perbaikan sebagian lapisan otot dan kulit d.Eksplorasi luka untuk memeriksa levator aponeurosis dan menempatkan kembali jika terlepas dari tarsusnya,sesudah perbaikan kulit dan otot e.Perbaikan laserasi kulit dan palpebra beberapa hari untuk merangsang penyembuhan. Jawaban : D 10. Pernyataan berikut yang salah mengenai Rhabdomyosarcoma adalah : a. b. c. d. e. Rata-rata umur 7-8 tahun Progresif proptosis unilateral Gejala yang muncul adalah: palpebra edema dan perubahan warna, ptosis, Hifema, Strabismus Kadang-kadang ada riwayat trauma Dapat metastase ke kelenjar limfa cervical dan preaureculer

Jawaban : E 11. Yang termasuk faktor resiko terjadinya Basal Cell Ca adalah : 1.Kulit kuning langsat, mata biru, rambut merah 2.Usia pertengahan ke atas 3.Exposure sinar matahari 4.Merokok Jawaban : E 12. Tanda-tanda okuler pada tumor limphoma maligna adalah: a. b. Specifik uveitis Umumnya bilateral

c. d. e.

Retinal vasculitis Vascular oclution Exudative retinal detachment

jawaban : A 13. Berdasarkan klasifikasi sitologi dari melanoma uvea, yang mempunyai prognosis paling jelek adalah : a.Spindle cell nevus b.Spindle Cell Melanoma c.Epithelioid melanoma d.Mixced cell type 14. Seorang pria Jepang, 41 tahun dengan riwayat trauma tumpul mata dengan visus yang baik dan tidak ada riwayat operasi sebelumnya. Keluhan saat ini terjadi penurunan visus dan nyeri berat pada kedua mata. Terdapat bilateral uveitis, alopecia, vitiligo dan kelainan serebrovaskular. Terdapat juga Retinal Detachment pada satu mata. Diagnosa yang tepat adalah a. Sarkoidosis b. Simpatetika oftalmika c. Vogt-Koyanagi-Harada Syndrome d. Bechet Syndrome Jawaban : C 15. Seorang perempuan umur 67 tahun dengan uveitis moderate dan vitritis dan subretinal infiltrat.Kondisi ini memberikan respon minimal terhadap topical kortikosteroid. Pasien merasa lemah dan pusing, test apa yang penting dilakukan : a. Gallium scan b. Westergren sedimentation rate dan C-reaktif protein c. PPD dan chest x-ray d. CT scan atau MRI kepala Jawaban : D 16. Organisme mana dibawah ini yang paling sering menyebabkan endoftalmitis ec trauma tetapi jarang menyebabkan endoftalmitis ec operasi katarak atau bleb related endoftalmitis :

a. Stap. Epidermididis b. Stap. Aureus c. Hemophyluss influensa d. Bacillus Sereus Jawaban : D 17. Seorang pasien dengan post trabekulektomi disertai mitomisin C, didapatkan severe bleb-related endoftalmitis. Visus awal 20/20 dan sekarang menjadi 1/300. Dimana pernyataan dibawah ini yang tidak tepat : a. b. c. d. Prognosis visus jelek Organisme yang berhubungan dengan bleb-related endoftalmitis adalah injeksi Hemofilus influensa atau Streptococcus. antibiotik Oleh karena visus lebih baik dari 1/~ diperlukan kultur vitreus dan Endoftalmitis bisa timbul berbulan-bulan sampai bertahun setelah operasi filtering glaukoma Jawaban : C 18. Seorang laki-laki kulit putih, 80 tahun dengan keluhan visus mata kanan menurun, nyeri dan hiperemi konjungtiva. Pada pemeriksaan ditemukan TIO 45 mmHg dengan adanya sel dan flare tanpa keratit presipitat. Terdapat juga katarak yang padat dan glaukoma sudut terbuka. Diagnosis yang paling tepat adalah a. b. c. d. Phacolitik glaukoma Phacoanafilaksis ICE sindrome Fuchs heterochromic iridosiklitis

Jawaban : A 19. Pada Persistent Corneal Epithelial Defect terdapat hal-hal berikut KECUALI : a. b. c. d. Defek kornea yang chronic non healing epithelium Hipostesi kornea Neovaskularisasi dan kekeruhan kornea Prediksi pada superior atau superior kornea

e.

Dapat terjadi perforasi kornea

Jawaban D (BSC 8 hal 96) 20. Phaco emulsification for cataract case with limited zenular support, the following factor must be remember : 1. 2. 3. 4. Reducing flow rate Insertion capsular tension ring Lowering the bottle height Increasing power ultrasound

Jawaban : A (BCSC sec 11 hal 215) 21. A child with anisometropic should be given: A. Full cycloplegic refractive the two eyes B. Face uncycloplegic refractive the twol eyes C. Under correction for the hight myopic eye D. Under correction for the hight myopic eye E. Over correction for lower myopic eye Jawaban : A (BCSC sec 3 hal 150)

22. Eye of the patients have not image magnification of minification like emetropic eye: A. Hyperopic eye with contact lens correctin B. Hyperopic eye with convex lens spectacle C. Myopic eye with contact lens correction D. Myopic eye with concave lens spectacle E. Myopic pseudophakia with concave lens spectacle Jawaban : E (BCSC sec 3 hal 209) 23. a. b. c. d. Retinal image of contact lens is influenced by : Power of contact lens Diameter of contact lens Refractive status of patients Verses distance of contact lens

e.

Anterior surface curvature of contact lens

Jawaban : D (BCSC sec 3 hal 173) 24. To reduce aniseikonia of two eyes, can be achived by the following may, exception : a. b. c. d. e. Miopic phakic eye be corrected by contact lens to increase mirage size Miopic phakic eye be corrected by contact lens to decrease mirage size Miopic phakic eye be corrected by spectacle lens to increase mirage size Hyperapic phakic eye be corrected by spectacle lens to decrease mirage size Hyperapic phakic eye be corrected by contact lens to increase mirage size Jawaban : A (BCSC sec 3 hal 175) 25.Tranposition of the lens S 3 = C + 3 x 70: a. S + 3 = C + 3 x 160 b. S 3 = C 3 x 70 c. C 3 x 160 d. C + 3 x 70 e. S + = C 3 x 160 Jawaban : C (BCSC sec 3 hal 92) 26. Pemeriksaan angiografi floresin pada choroidal neovascularization memeprlihatkan gambaran hiperfloresin pada fase ; a. b. c. d. e. prearterial arterial aretrivenous venous last phase. AAO, Sec 12, 14, 21. AAO 2003-04. 27. a. Penyebab tersering gagalnya penanganan operasi dari ablasi retina giant retinal break

Jawaban A.

rhegmatogen adalah karena adanya :

b. c. d. e.

proliferative vitreoretinopathy lattice degeneration lebih dari 2 kuadrant perdarahan vitreous Posterior vitrewous detachment

Jawaban : B AAO, Sec 12, 246, AAO 2003-04. 28. Seorang anak umur 3 tahun, dengan kataran pada mata kiri, menunjukkan

fiksasi yang jelek dan esotropia di mata kiri. Mata kanan nampak normal. Manakah kalimat dibawah ini yang paling tepat : a. b. c. d. e. Terapi ambliopia harus diberikan mengawali operasi katarak Kapsulotomi posterior tidak boleh dilakukan pada saat operasi oleh karena dapat menyebabkan ablasi Operasi katarak dengan implantasi IOL dapat dilakukan untuk merehabilitasi visus Implatasi IOL tidak boleh dilakukan pada anak-anak Esotropia mata kiri harus diperbaiki mengawali operasi katarak

Jawaban : C

29.

Seorang wanita, 55 tahun dengan miop dan keluhan diplopia monokular

disertai keluhan sulit mengendarai mobil pada malam hari. BCVA -2 D, Visus 20/30. Pada pemeriksaan slit lamp terdapat nuklear sklerosis minimal. Pemeriksaan tambahan apa yang diperlukan untuk mengevaluasi gejalanya : a. b. c. d. Topografi kornea Red refleks MRI Fluorosens angiografi

Jawaban : B

30.

Seorang laki-laki, 70 tahun dengan keluhan kabur saat mengendarai mobil.

Visus 20/70 mata kanan, dan 20/40 mata kiri. Test lapang pandang Goldman

terdapat konstriksi pada mata kanan.Terdapat juga nuklear katarak yang moderat pada mata kanan dan yang ringan pada mata kiri. TOD 23mmHg, TOS 18mmHg. Pasien menggunakan timolol dan dorzolamide pada mata kanan. CDR OD 0,8, OS 0,6. Pemeriksaan fundus lainnya dalam batas normal. Pernyataan yang benar adalah : a. b. c. d. e. Operasi katarak disertai operasi filtrasi glaucoma adalah satu-satunya terapi Penggunaan latanoprost sesudah operasi katarak dapat meningkatkan resiko post op CME Operasi katarak tidak dapat dilakukan karena resiko kehilangan fiksasi post op katarak Obat-obat glaukoma harus diberikan secara maksimal sebelum operasi katarak Konstriksi pada pemeriksaan lapangan pandang kemungkinan disebabkan oleh Jawaban : B 31.Yang mana pengobatan yang paling tepat untuk bayi tersebut : a. mata yang terbaik fiksasinya ditutup dan diikuti dengan operasi pada mata yang lain b. Segera lakukan operasi pada satu mata, dan mata yang lain pada umur 3 bulan c. Lakukan operasi pada satu mata dan segera lakukan koreksi afakia, lakukan operasi pada mata sebelah sebelum umur 3 bulan d. Observasi sampai umur 3 bulan dan lakukan operasi apabila timbul nystagmus e. Lakukan operasi pada kedua mata pada umur 3 bulan Jawaban : C (BCSC 11, hal 195) 32. Pada bayi tadi metode operasi yang paling tepat atau tahapan selam operasi katarak adalah sebagi berikut KECUALI a. Aspirasi lensa yang katarak b. Implantasi posterior chamber IOL glaukoma

c. Kapsulotomi posterior dan vitrektomi anterior secukupnya d. ECCE e. Small Incisisi Cataract Surgery Jawaban : B (BCSC 11 hal 156) 33. Seorang laki-laki umur 60 tahun, berhasil dioperasi nuclear sklerotik katarak pada mata kanannya dengan fekoemulsifikasi. Dokter bedah mata secara tidak sengaja meninggalkan viscoelastik yang cukup banyak didalam mata. Berapa lama setelah operasi selesai terjadi peningkatan TIO secara signifikan. a. 30 menit b. 2 jam c. 4 jam d. 10 jam e. 24 jam Jawaban : C (BCSC 11, hal 172) 34. You have +10D lens dan +20D lens. You want to make a 2x magnifier. What is the distance between the lenses to produce this magnification? a. 10 cm b. 15 cm c. 5 cm d. 25 cm Jawaban : C (BCSC 3,hal 57) 35. You have two +2,00 D lenses. How far apart are the two lenses so that an object at infinity is focused 1m to the right of the second lens? a. 1.0m b. 0.5m c. 1.5m

d. 0.75m Jawaban : C (BCSC 3, hal 57) 36. A Galielan telescope is constructed with a +5,00D objective and a -10,00D eyepiece. What is magnification of the telescope? a. X b. 50 X c. 2 X d. 5 X Jawaban : C (BCSC 3, hal 74) 37. Seorang wanita 44 tahun, datang dengan keluhan diplopia. Ptosis 4 mm, tidak ditemukan upgaze dan adduksi, serta 50% reduksi pada abduksi. Pupil 5 mm dan kurang reaktif, tetapi tidak terdapat RAPD. Visus normal, pada hasil pemeriksaan eksterna dan pemeriksaan fundus normal. Lokasi lesi yang sering ditemukan adalah a. Orbit b. Sinus cavernosus c. Brain stem d. Neuromuscular junction Jawaban : B 38. Sehubungan dengan pertanyaan diatas, prosedur pertama yang terbaik adalah a. MRI dengan kontras b. CT scan tanpa kontras c. USG d. Tes Tensilon Jawaban : A 39. MRI memberikan hasil yang lebih baik dibanding CT

scan kecuali pada keadaan dibawah ini a. Penyakit demielinisasi b. Infark akut c. Hemorrahagic akut d. Abnormal parenkim kongenital Jawaban : C 40. Seorang wanita obesitas, 23 tahun, datang dengan keluhan sakit kepala dan transient visual obscuration. Terdapat papil edem bilateral. Langkah selanjutnya adalah a. Dimulai dengan pemberian acetazolamide oral b. Pemeriksaan Lumbal puncture c. Menganjurkan diet d. Pemeriksaan CT scan dan MRI Jawaban : D

TRY OUT BOARD UNSRAT DESEMBER 2007


1. 30 years old male came to you with redness on his left eye since 2 days ago. The redness of

the eye occurred since he had trauma from a branch of three. On slit lamp examination : little conjunctival injection, gray white, dry appearing corneal infiltrate with feathery margins. Corneal scrapping reveals fusarium spp as the infectious cause. The treatment for this patient is : a. Miconazole 1% b. Oral fluconazole 200-400 mg/day c. Chlorhexidine topical 0,02% d. Propamidine isethionate 0,1% e. Natamycin 5% Answer E 2. A 25 years beautiful girl came to your clinic with redness of the eye for 2 months. Examination reveals follicle in superior tarsus conjunctiva, diffuse papillary hypertrophy, artls line and herberts pits. These statements are true : 1. 2. 3. Isolation the causative agent with McCoy cells Topical tetracycline 1% twice daily for 2 months Resolve spontaneously in 6-18 months if left untreated

4. Oral erythromycin 500 mg 4 times a day for 3 weeks Answer E 3. A baby 3 days old come to eye clinic with complain copious purulent conjunctival discharge, marked conjunctival hyperemia and chemosis since 2 days ago. The appropriate diagnosis for this patient are true : a. Gonoccocal conjunctivitis b. Chlamydial conjuctivitis c. Microsporidial conjunctivitis d. Trachoma e. Loisiasis Answer A 4. 54 year old woman complains of discomfort and foreign body sensation in her right eye over the past few months. Slit lamp shows papillary hypertrophy of the upper palpebral conjunctiva with prominent vessels and hyperemia of the upper bulbar conjunctiva with punctate staining with rose bengal. Corneal changes reveal micropannus and fine filaments at the superior periphery. The most likely diagnosis is:

a. Terriens degeneration b. Moorens ulcer c. Superior limbic keratoconjunctivitis d. Trachoma e. Rheumatoid arthritis with ocular involvement Answer C 5. Streak retinoscopy was perfomed on a 3 years japanese old boy. Working distance was 67 cm. You find with the movement sweeping the 180 meridian and against movement sweeping 90 meridian. The power cross is : What is the correct spectacle prescription? a. +3.50-5.00x180 b. -3.50+1.50x180 +3D c. -2.00 +5.00x180 d. +1.50 -5.00x180 e. +1.50 -5.00x90 Answer D 6. A 4 year old child is referred for a new onset of bilateral epiphora. Examination shows on both lower eyelids rubbing against the inferior cornea. The parents state that similar symptoms occurred in an older sibling but resolved without treatment. What is the most likely diagnosis:

-2D

a. Epiblepharon b. Entropion c. Euryblepharon d. Trichiasis e. Ankyloblepharon Answer A


7.

What is the pathophysiologic mechanism underlying the condition in above question :

a. Laxity of tarsal plate b. Laxity of the canthal tendons c. Abnormal attachment of the orbital septum d. redudancy of skin and pretarsal muscle e. none of above Answer D 8. 50 year old man came to the eye clinic with the chief complain of pain on his right eye since 3 days ago, pain associated with tearing, edema and erythema within his medial canthal regio with distension of the lacrimal sac. These are true except : a. b. c. d. e. Therapeutic probing of the nasolacrimal duct with Bowman probe Topical antibiotics are limited value when stasis is present Aspiration of the lacrimal sac if the mucocele/pyocele is localised and pointing A localized abscess requires incision and drainage, the incised abscess is packed open Gram positive bacteria is the most common cause

Answer A 9. A 60 years old woman had headache, red eye, microcystic corneal edema following cataract extraction in the left eye 2 days ago. The treatment of choice in these case are: a. Parasympathomimetic agent b. Corticosteroids and carbonic anhidrase inhibitors c. Prostaglandin analogues d. Cholinergic agonists e. Anticholine esterase agents Answer: B.
10.The diagnosis in above Question is :

a. Phacolytic glaucoma b. Phacoanaphylaxis glaucoma c. Phacomorphic glaucoma d. Lens particle glaucoma e. Ectopia lentis Answer: D. 11. A 60 years old black man came in for an eye examination. He did not have any specific complaints. He uses only over the counter reading glasses and had never visited an ophthalmologist in the past. Visual acuity was 20/60 OU with correction. The anterior segment of both eyes was normal with the exception of early cataractous changes OU. The IOP was 30 mm Hg OU. What kind of further examination must be done to diagnose this complaints:

1. Opthalmoscopy 2. Gonioscopy 3. Visual field test 4. Contrast sensitivity test Answer: A. 12. A 15 year old student was brought to the Clinic with a history of trauma to the left eye while playing football. The visual acuity was hand movement OS and 20/20 OD. Examination of the left eye showed circumcorneal congestion, a moderately haze cornea, and small haemorhage in AC. The IOP was 50 mm Hg. Gonioscopy reveals angle recession. The treatment is often best accomplish with, except: Timolol Maleat 0.5% Acetazolamide Laser Trabeculoplasty Apraclonidin HCl Latanoprost Answer: C. 13. A 56 year-old woman presented to the eye clinic with sudden blurred vision, headaches sometimes her eye feel mild pain and see halos. The pain and blurred vision resolve spontaneously, especially during sleep-induced miosis. She routine came to ophthalmologist with the IOP sometimes raised and sometimes normal. The visual acuity was 6/60 OD and 20/20 OS. The right eye showed circumcorneal injection, mild diffuse corneal haze, shallow anterior chamber and the pupil slow adapted to the light. The lens was early cataractous changes. The left eye was normal except for a shallow AC. The IOP was 40 mm Hg OD and 16 mm Hg OS. Gonioscopy OD revealed narrow angle and PAS at 11-12 oclock. Gonioscopy OS revealed a narrow angle. The Diagnosis is: a. Primary Angle-closure Glaucoma b. Chronic Angle Closure c. Intermitten angle Closure d. Primary Open Angle e. Phacomorphic glaucoma Answer: C.
14.Treatment of choice in this case is:

a. b. c. d. e.

a. Glycerin oral b. Pilocarpine Hcl 10% c. Timolol Maleat 0.5% d. Acetazolamide oral e. Laser iridectomy Answer: E 15. A 50 year old man complaining glare especially under bright lightning condition. His near visual acuity is worse than distance visual acuity, even after correction for presbiopia. In slit lamp examination you saw this : Your diagnosis :

Figure 1

a. b. c. d. e.

Anterior polar cataract Cortical cataract Posterior subcapsular cataract Morgagnian cataract Hypermature cataract

Answer : C 16. If this patient were treated with topical pilocarpine, he will experience : 1. better near vision 2. worse near vision 3. better far vision 4. worse far vision Answer : C 17. This kind of cataract in above question can be caused by : 1. Senescent 2. Trauma 3. Inflammation 4. Topical corticosteroid use Answer : E 18. A patient is noted to have nuclear sclerotic cataract. Cataract of this type may be associated with all the following except: a. Early loss of near (reading) vision. b. A myopic shift. c. Monocular diplopia. d. Difficulty seeing road signs at dusk. e. Loss of far vision. Answer A 19. On pupillary dilation, a patient shows the characteristic lens change inferiorly dislocated lens with equatorial zonular remnants. The least likely diagnosis is: a. Marfan syndrome b. Homocystinuria c. Weill-Marchesani syndrome d. Sturge-Weber syndrome e. Hyperlysinemia

Answer D 20. A 25 years old man came to the eye clinic. Ophthalmologic examination revealed a golden brown discoloration of Descemets membrane around the periphery of the cornea, and sunflower cataract. The predisposing disease is : a. Galactosemia b. Myotonic dystrophy c. Hypocalcemia d. Hepatolenticular degeneration e. Infrared radiation Answer D 21. A 65 year old man complains of difficulty driving because of reduced vision. His bestcorrected visual acuity is 20/70 OD and 20/40 OS. Goldman visual fields are constricted, more in the OD than in the OS. A moderate nuclear cataract is present OD, and mild one is seen OS. His IOP is 23 mm Hg OD and 18 mm Hg OS. He uses timolol % bid OD and Dorzolamide tid OD. His cup- disc ratio is 0.8 OD and 0.6 OS. Which of this following statement is true? a. Cataract surgery should not be considered because of the risk of loss of fixation postoperatively b. Cataract surgery combined with glaucoma filtering surgery is the only approach that should be considered for this patient c. The use of latanaprost after cataract surgery may increase the risk of postoperative CME d. Staged procedures is a cataract surgery followed by filtering surgery e. The visual field constriction in this case is probably caused by glaucoma Answer: C. 22. A two weeks old baby boy came with bilateral congenital cataract. The TORCH examination is negative. The proper time for the cataract extraction is : a. under 3 month for the first eye, and 1 month later for the second eye above 3 month for the first eye, and 2 weeks later for the second eye b. under 3 month for the first eye, and 2 weeks later for the second eye c. above 3 month, together both eyes d. under 1 year for the first eye, and 1 month later for the second eye Answer: C 23. Two years after the operation was done, his parents want to correct their babys aphakic condition. The ophthalmologist suggests secondary IOL implantation. Which is the true statement : Fixation in the sulcus is preferable to fixation in the capsular bag Use the + 20 D IOLs, without calculate the intraocular power Anterior chamber IOLs are generally recommended There is an improved of binocularity when compared with contact lenses Bilateral lens implantation surgery in children above 2 year of age is still controversial Answer D. 24. Post operative, this baby should receive one of these, except : a. b. c. d. e.

a. Fluometholone and Scopolamine 0,25% for about 1 month b. Prednisolone and Phenilephrine 0,1% for about 1 month c. Hydrocortisone and Cyclopentolate 1% for about 1 month d. Betamethason and Atropine 1% for about 1 month e. Dexamethason and Cyclopentolate 2% for about 1 month Answer B. 25. A 25-year-old woman notes the acute onset of painless photopsias and a blind spot in the temporal field of her left eye. Her visual acuity and color vision are normal. On visual field examination, her left blind spot is three times the normal size. Fundus examination shows multiple, faint, tiny spots in the posterior pole OS. The examination is otherwise entirely normal. Examination 3 days later shows the spots to be gone. Which of the following diagnosis is the most likely? a. Pseudotumor cerebri b. Acute posterior multifocal placoid pigment epitheliopathy c. Acute zonal occult outer retinopathy (AZOOR) d. Optic neuritis e. Canthaxanthine maculopathy Answer: C 26. A 24-year-old man complains of episodes of blurred vision with flashing lights that progress over approximately 30 minutes. There are followed by a throbbing headache and resolution of his visual aymptoms. A complete ophthalmologic evaluation, including visual fields, is normal. The most likely diagnosis is a. Cluster headache b. Classic migraine c. Transient ischemic attack d. Superior oblique myokymia Answer B 27. A 8 years old girl presents with a left upper eyelid edema and erythema. According her mother this condition has been presents for 2 days without history of trauma. On examination revealed her vision 20/20, pupillary reaction was normal, proptotic is not presents and no limitation of ocular motility. Pain on movement of the globe is absent. The most likely diagnosis is : a. b. c. d. e. Orbital cellulitis Preseptal cellulites Idiopathic orbital inflammation Rhabdomyosarcoma Carotid cavernous sinus fistula

Answer B 28. Which of the following statement about above condition is false: a. Commonly occurs in children b. The most frequent cause is ethmoidal sinusitis c. Imaging studies should be performed d. Respond to single antibiotic theraphy e. Involvement of the orbital apex

29.

Answer E A 70 years old actrist presents complaining of droopy eyelids, superior fields defect and difficulty for reading. Examination reveals pupillary reaction was normal and marginal reflex distance is found to be 1 mm. A tensilon test was negative. The most likely diagnosis is :

a. Myastenia gravis b. Dermatochalasis c. Cranial nerve III palsy d. Blepharoptosis e. Horners syndrome Answer D 30. Which of the following in the clinical evaluation is most significant to compare acquired type than congenital type of diagnosis in above question ? 1. Upper eyelid crease 2. Levator function 3. Eyelid displacement on downgaze 4. Palpebra fissure height Answer A 31. What is pathophysiologic mechanism underlying the condition in above question ? a. Muscle dysfunction b. Neurogenic pathology c. Aponeurotic dysfunction d. Redudancy of skin e. Orbital fat prolapse Answer C 32. A 9-month-old baby girl present to the eye clinic for crossed eye. She seems always to have both eye crossed inward, all of the time, regardless of whether she is looking at near or at distance. Birth history was normal, she is in good physical health, with fix and follow behavior, normal anterior and posterior segment of both eyes. The corneal light reflex test shows equal and a cover test show no movement. The eyes appear to be crossed inward, with less sclera showing in the nasal portions of the eyes as compared with the temporal portions. The most likely diagnosis is: a. Infantile esotropia b. Incomitant esotropia c. Pseudoesotropia d. Basic esotropia e. Sensory deprivation esodeviation Answer C 33. You are asked to see a 5-year-old little girl with amblyopia. How do you most likely manage her ? 1. 2. 3. 4. Obstacle removal Refractive correction Occlusion and optical degradation Kestenbaum-Anderson Procedure

Answer A 34. A mother complains that her 1-year-old boy sometimes has a visual inattention. His left eye first noticed by his mother as a glint, white to cream colored. The first examination notes leukocoria on his left eye. The least likely differential diagnosis of this boy is: 1. Persisten hyperplastic primary vitreus 2. Chorioretinal colobomas 3. Retinoblastoma 4. Brushfield spots Answer D 35. A healthy 30 yo man presents with a 1-week history of pain, redness and floaters in his left eye. Examination reveals VA of 20/25 OS. Significant finding on examination are mild ciliary injection, fine keratic precipitates, mild anterior chamber cell, clear lens, and moderate anterior vitreous cell. The posterior pole appears normal except for a slightly hazy view secondary to vitreous debris. The lesion shown in the peripheral retinal. What is the most likely diagnosis ? A. Cytomegalovirus retinitis (CMV) B. Toxoplasmic retinochoroiditis C. Endogenous bacterial endophthalmitis D. Behcet syndrome E. Acute retinal necrosis (ARN) Answer E. 36. The most appropriate initial treatment for the patient in above question is : A. Intravenous acyclovir for 10-14 day follow by oral valacyclovir, famcyclovir or acyclovir for up 3 months B. Intravenous ganciclovir induction for two weeks, followed by daily maintenance ganciclovir C. Oral prednisone 60mg to 80 mg perday D. Erythromycin (500mg qid) or ciprofloxacin (750mg bid) E. Laser photocoagulation to demarcate the borders of retinal necrosis Answer A. 37. An 40 yo man was playing card when he noticed the vision become dim in his left eye. He felt well otherwise and had no pain. After a few minutes, he told his companions, and they brought him to an emergency room. Vision was limited to light perception OS, but OD was 20/25. There was a left afferent papillary defect . The fundus OS showed narrowed vessels with boxcarring. Most of the retina was abnormally pale, and the macula showed a cherry red spot. Which statement is correct for this diagnosis ?

1. Fluoresencein angiography can also be helpful in diagnosis because many patients exhibit a phenomenon dark choroids 2. Irreversible damage to the sensory retina occurs after 90-100 minutes of complete disease 3. Other treatments advocated include inhalation therapy with 25% oxygen-65% carbon dioxide mixture 4. It is often caused by atherosclerosis-related thrombosis occurring at the level of the lamina cribosa.

Answer C. 38. A 40 yo man was reffered to ophthalmologist from a neurologist with clinical findings cerebellar ataxia,polyneuropathy,anosmia,deafness. An ECG examination showed arrythmias. He was reffered because prior of those disorders, he complain about difficulty to see in dusk. In the examination retinal athrophy and spiculed pigmentary degeneration was founded. The diagnosis of this condition : a. Mucopolysaccharidoses b. Tay-sachs disease c. Kearns-sayre syndrome d. Refsum disease e. Bardet-Biedl syndrome Answer D 39. A 30 years old pregnant women was suspected with ocular toxoplasmosis. The anterior segment was normal with whitish yellow, slightly raised, fuzzy lesion with a chorioretinal scar on funduscopic examination. IgM antibody titer for toxoplasma was positive. Right now, she is 5 months pregnant. What is the treatment for this patient? a. Pyrimethamine & Sulfonamide b. Clindamycin c. Spiramycin & sulfadiazine d. Atovaquine e. Spiramycin & Pyrimethamine Answer C

40. A 40 yo man was reffered to ophthalmologist from a neurologist with clinical findings cerebellar ataxia,polyneuropathy,anosmia,deafness. An ECG examination showed arrythmias. He was reffered because prior of those disorders, he complain about difficulty to see in dusk. In the examination retinal athrophy and spiculed pigmentary degeneration was founded. The diagnosis of this condition : a. Mucopolysaccharidoses b. Tay-sachs disease c. Kearns-sayre syndrome d. Refsum disease e. Bardet-Biedl syndrome Answer D. --good luck--

EDIT 1. A 12 year old man came to emergency room with main complain the eye lid edema since 4 days ago. Examination result with sinusitis, eyelid edema, erythema and inflammation reaction around the orbita. VA 6/6, pupillary reaction, and ocular motility are not disturbed. The globe are not involved. Pain on eye movement and chemosis are absent. What is the diagnosis in this case? A. Preseptal Cellulitis B. Orbital Cellulitis C. Panophthalmitis D. Necrotizing Fasciitis E. Aspergilosis 2. A 20 year old man came to emergency room with main complain eye lid edema and fever since 7 days. Examination result with proptosis, chemosis, sinusitis, eyelid edema, erythema and severe inflammation. VA 6/60, pain movement of the globe, and restriction on ocular motility .The UCG Examination normal laboratory has a leukositosis.What is the diagnosis in this case? A. Preseptal Cellulitis B. Orbita l Cellulitis C. Endofthalmitis D. Necrotizing Fasciitis E. Pan Opthalmitis 3. A 24 year old smoker present with painless, semiacute, progressive visual loss in his right eye. Fundus both eyes and MRI are normal. Three months later, the affected eye has not been improved yet, and the vision has deteoritated. Repeated MRI and lumbar puncture

yield negative results. The complete blood count and assay for antinuclea antibody, fluorescent treponemal antibody absorption, angiotensin convertingen enzyme, lyme titer, vitamin B 12, and folate are normal. What is the most likely diagnosis ? A. Multiple sclerosis (MS) B. Tobacco alcohol optic neuropathy C. Posterior ischemic optic neuropathy (PION) D. Lebel heriditary optic neuropathy (LHON) E. Anterior ischemic optic neuropathy 4. A 10 years old boy is examined for bilateral progressive visual loss that occurred over 1 month. He has polyuria and polydipsia and is found to have diabetes insipidus. Which of the following is the most likely associated visual field deficit ? A. Unilateral nasal step B. Bilateral nasal defect respecting in the midline C. A complete left homonymous hemianopsia D. A bilateral temporal defect respecting the midline, denser in the inferior quadrants E. Altitudinal 5. A 40 years old male came to policlinic with main complain pain, redness, haloes left eye. Previous history illness : no flu like syndrome. The doctor examined and found elevated IOP on the left eye, corneal edeme, fine KPs, low grade cell dan flare in anterior chamber a slightly dilatated pupil , no sinechia, no vitreus cell . What is the diagnosis in this patient ?. A. Posner Schlossman Syndrome B. Phacolytic Glaukoma C. Fuchs Uveitis Syndrome D. Voght Koyanagi Harada desease E. Exfoliation syndrome 6. AAO Sect 9 page 135

A. 22 years old female came to policlinic with main complain photopobia and decreased visual aquity both eyes. Ophthalmological examination found visual aquity R-LE 6/20 PH( - ) , KPs (+) and snowbank on the inferior pars plana. Retinal venous sheating at the peripheral retina and no cystoid macular edema. Patient is often complain pain in her teeth and dentist found dental carries. What is the diagnosis in this patient ? A. Anterior Uveitis B. Intermediate Uveitis C. Posterior Uveitis D. Pan Uveitis E. Pars Planitis AAO Sect 9 page 147

7.

Young man came to RSMH with main complain diplopia, the RE deviated medialy and stabilized. From examination we found that the patient head positionn is normal, limitation of abduction, VA equal both eyes. Trauma history a year ago and no family history of strabismus, no proptosis and redness. Force duction test no restriction. Which one is the weakening procedures used in strabismus surgery? a. removal and reetachment of a medial rectus muscle , insertion is closer to its origin b. absorbable sutures of the medial rectus muscle are placed at predetermined distance posterior c. the muscle of the medial rectus muscle anterior to the position of the sutures is excised d. the shortened medial rectus muscle is reattached to the globe at or near the original insertion e. advancing the insertion of the medial rectus muscle nearer the limbus

8.

A 4 months girl came with her mother with complain RE going medialy. The eye were alignment, and no history of trauma. Examination: VA exist, normal head position, limited

normal at birth with good horizontal rotation, primary position esotropia and poor abduction. The laboratory normal. Forced duction test: restriction What is the diagnosis of this patients? a. b. c. d. e. 9. congenital fibrotic exstraocular muscle duane syndrome graves desease myasthenia gravis mobius Syndroma

A 40 years old Man came to policlinic with a refraction disorder / astigmatism, and he has a pair of soft contact lenses for his eyes. What is the most common adverse reaction to soft contact lenses ? a. b. c. d. e. Giant papillary conjungtivitis (GPC) Corneal neovascularization Corneal hypoxia Conjungtivitis Keratitis AAO section 3, 197 199

10.

A man, 23 years old wears rigid / hard gas permeable (RGP) contact lenses, in examination the position is low riding, then inadequate tear meniscus, insufficient lens

movement, thick peripheral lens profiles and abortive reflex blink patern. What is the possible adverse reaction ? a. b. c. d. e. 11. 3 & 9 oclock staining Lens adhesion Ptosis Spectacle blur Miscellaneous AAO section 3, 194

Bacterial infection of the orbit or periorbital soft tissues occurs from primary sources : 1. Direct spread from an adjacent sinusitis 2. Direct innoculation following trauma 3. Bacteremic spread from a distant focus 4. Direct innoculation following skin infection AAO 7, 41

12.

A girl, 3 years old came to emergency with main complain eyelid edema since 7 days. Ophthalmology examination VODS: fixation (+), eyelid edema, pupillary reaction (+), ocular motility are not disturbed. Pain on eye movement and chemosis are absent. Ophthalmologist diagnosed this case with preseptal cellulitis. Historically preseptal cellulitis in infant and children was often associated with, 1. Bacteremia 2. Septicemia 3. Cutaneous infection 4. Meningitis and sinusitis 13. A 40 year old Man had basal carcinoma in the upper eyelid. He had surgery and after that he had a 40% upper lid defect. What kind of repair needed to solve this case? a. Primary closure with lateral canthotomy b. Primary closure with superior cantholysis c. Adjacent tarsoconjunctival flap and full-thickness skin graft d. Free tarsoconjunctival graft and skin flap e. Full thickness lower eyelid advancement flap AAO 7, 190

14.

A 50 year old woman came to policlinic with main complain tearing. From examination, she had severe involutional ectropion and involves more than the punctum. What kind of surgical generally used for this case ? 1. horizontal resection of the lateral eyelid 2. plication of the canthal tendon 3. lateral canthoplasty 4. reinsertion of the lower eyelid retractor AAO 7, 197

15. A young man came hospital with his chief complain discomfort on his eyes, he has a dry mouth and history of remathoid arthritis. In minor glandula salivary biopsy found inflamatory infiltrate. What is the diagnosis of this patient? a. Sjogen syndrome b. SLE c. Scleroderma d. Reiter syndrome e. Polymyositis 16. A young man came to a hospital with main complain fever, chills malaise. Thyroid fuction test t4 level elevated and low RAIU. After resolution period this man got transient hipothyroidism. What is the man illness? a. Acute thyroiditis b. Sub acute Granulomatous Thyroiditis c. Sub acute Lymphocitic Thyroiditis d. Hashimoto Disease. 17. A man 20 years old came to hospital with main complain pain and red eye. Two weeks before, he had an injury from tree of rice, from examination mix injectie was profound, paracentral ulcus with hipopion in anterior chamber. Pseudomonas aurigenosa was found from the culture, what is the effective therapy : a. b. c. d. e. 18. Aminoglycoside Ceftazidine + Aminoglycoside Cyprofloxacine Imipenem and piperacilin / tazobactam Ceftazidine

A women, 30 years old consult from surgery department with complain diplopia, enofthalmus and hyposthesia in the distribution of infraorbital nerve. After examination a positive forced duction test, and radiographic evidence of a fluid level in the maxillary sinus. She has diagnose Blunt trauma to the tissues of the orbit may fracture of the fragile bony orbital floor. What is the bony orbital floor ? a. Frontal process of the maxilla, lacrimal, orbital plate of the ethmoid, lesser wing of the sphenoid b. Maxillary, palatine, orbital plate of the zygomatic c. Orbital plate of the frontal, lesser wing of the sphenoid d. Zygomatic, greater wing of the sphenoid e. Palatine, zygomatic and maxillary

19. A father brought his 2 days old boy to an eye policlinic ,whose born with some abnormality such as strabismus, blepharophitosis , epicantus, cataract, glaucoma, coloboma of uvea, persistent fetal vasculature, dismorphogenesis of retina, optic nerve hipoplasia. There was no family history of this disease.What kind of drug involved in this case? A. Cocaine B. Heroine C. Barbiturat D. Alcohol E. Tobacco 20. A young mother went to a hospital with chief complain of ocular pain, head ache, blurred vision, rainbow colored halo around light, nausea and vomiting. She has no history of hipertension, nor diabetes. What is the best procedure for a good definitif diagnosis about the patient: A. Fluoresence B. Keratometer C. Hertel D. Gonioscopy E. Tonometry 21. A 26 years old myopic man presents with a 5 day history of photopsia, small scotomas, and blurred vision both eyes. He is recovering from a rescent flu like illnness. Examination reveals best corrected visual acuity of 20/50 OD and 20/ 40 OS. Slit lamp examination show mild flare and cell in both anterior chambers and mild vitreous cell in both eyes. The fundus finding are similary in both eyes with the multifocal choroiditis and chorioretinal scars in the nasal retina. Which the following diagnoses is most likely in this patient ? a. Presumed ocular histoplasmosis syndrome b. Multifocal choroiditis c. Birdshot retinochoroidopathy d. Acute posterior multi focal placoid pigment epitheliopathy e. Posterior uveitis 22. A 32 year old man present without complaints for routine eye check up. On direct opthalmoscopy, multiple patches of peripheral laticce degeneration containing multiple atroptihc retinal holes are noted in the superior retina OD. Laticce degeneration

without retinal breaks is noted inferior OS. Which of the following statements regarding prophylactic treatment is most correct ? a. b. c. d. e. Only the patients right eye should be prophylactically treated with laser photocoagulation or cryoretinopexy Both of the patients eyes should be propylactically treated with laser photocoagulation or cryoretinopexy If there is a prior history of retinal detachement in the left eye, propylactic laser photocoagulation or cyroretinopexy should be considered in the right eye The patients right eye should receive prophylactic treatment with laser photocoagulation or cryoretinopexy prior to cataract surgery Informed the patient and do nothing

23. (Refraksi, AAO 3, 239) A girl, 20 years old came to ophthalmologic policlinic, she was diagnosed with profound low vision. Then she was consultated to rehabilitation department, for maximizing her low vision with training. She can develop skills to make effective use of her low vision. She refers to to the gathering of visual information by the movement of the eye or head. It is a. Scanning b. Tracing c. Spotting d. Tracking e. Blinking 24. (refraksi, AAO 3, 88) A girl came to ophthalmologic policlinic. She was diagnosed with simplex myopia and compositum astigmatism, the she was given trial lenses, a spherical and cylindrical spectacle lense at left and right eyes. When spherical and cilincdrical spectacle was used to gether at both eyes, she fell confuse, then the doctor used spherical equivalent of the lens. It is a. = sphere +cylinder 2 b. = sphere+cylinder 3 c. = sphere+cylinder 4 d. =sphere+cylinder 5

25.

A 29 year old man presents with a 4 week history of floaters and blurred vision in his right eye. Six weeks ago he developed intermittent daily headaches, lympadenopathy, fever, and right sweats all of which have improved within the last weeks. He has no history of prior medical problems, denies prior illicit drug use, is employed as a construction worker, and is an avid hunter who frequently eats game. Visual Acuity is 20/60 OD, and 20/20 OS, mild cell and flare is present in the right anterior chamber. The conjunctiva are notinjected, the corneas are clear, the lens is clear, and intraocular pressures are normal. Moderately fine vitreous cell is present in the right eye, the vitreous in the left eye is clear.fundus right eye focal retinochoroiditis in posterior pole. The left is normal. Which of the following diagnoses is most likely in this patient? a. Acute retinal necrosis b. Toxoplasma chorioretinitis c. CMV (cytomegalovirus) retinitis d. Lyme disease e. Sarcoidosis p.186

26.

A 27 year old man presents with a history of floaters and blurred vision OD. The patient was immigrated from India within the last year, is currently good health, and denies any history of drug use. Two years ago, while in India, he had a positive tuberkulin skin test and received 9 moths of prophylactic INH, Rifampin, and Ethambutol therapy. Anterior segment is normal. The right and left fundus is show retinal neovascularization , Vitreous hemorrahage is noted in the inferior vitreous OD. No pars plana snowbanks are noted in either eye. Which of the following diagnoses is most likely in this patient ? a. Eales disease b. Sarcoidosis c. Sickle cell retinopathy d. Cat-scratch disease e. ARN p. 153

27. patient?

A 30 years old man came to policlinic and diagnosed with ptosis. What should we examine in this 1. vertical interpalpebral fissure height 2. margin-reflex distance 3. upper eyelid crease position 4. levator function

28. A 30 years old man came to policlinic and diagnosed with ptosis. The most commonly surgical procedures that use in this patient are . 1. external levator advancement 2. frontalis muscle suspension 3. internal levator/tarsus/Muller`s muscle resection approaches 4. horizontal eyelid tightening 29. A Child 10 years old came to hospital with her mother with main complain blured vision right eye, red eye (+), pain (+). He has trauma 2 weeks ago in examination VOD 6/60, VOS 6/6 injectie conjunctiva (+), hifema COA. He has body weight 25 kg. He must stay in hospital and get Tranexamat acid and Elevation of head 45 o. How much dose of Tranexamad acid for that patient ? a. b. c. d. 30. 250 mg / day 375 mg / day 500 mg / day 625 mg / day AAO 2 hal 453

A Child, 8 years came to hospital with main complain proptosis since 1 year, red eye (+), pain (+), in examination leukokoria (+), eye like cat (+). He was diagnosed with susp retinoblastoma. What is the predosposition factor for retinoblastoma? a. b. c. d. e. Translocation chromoseme 13q14 Mutation chromosone 14 Delesi chromoson band long arm 13q14 Delesi chromosom short arm 13p14 Mutation chromosome 13 AAO 2 hal 253

31. A father brought his 2 days infant to eye policlinic ,whose born with some abnormality such as strabismus, blepharoptosis , epicantus, also cataract, glaucoma, coloboma of uvea, persistent fetal vasculatur, dismorphogenesis of retina, optic nerve hipoplasia. There was no family history of this disease. What kind of drug involved in this case? A. Cocaine B. Heroine C. Barbiturat D. Alcohol E. Tobacco

3. 3In the evaluation and treatment of Diabetic Retinopathy, FFA is useful:

a. b. c. d. e

To determine areas of capillary closure To determine the absence of CSME prior to recommending photocoagulation To determine the location of diffuse retinal leakage from incompetent retinal capillaries or IRMA prior to photocoagulation To determine the location of retinal microaneurysms prior to photocoagulation To determine IRMA AAO 1 2 hal 114

34. A man 32 year old with a history of AIDS and CMV retinitis initially received ganciclovir but was switched to foscarnet 6 months later when recurrence of the CMV retinitis indicated resistance to Ganciclovir. Following forcarnet therapy, all CMV activity in the retina resolved. The patient now presents with the fundus recurrent CMV in the nasal retina on the left eye. His creatinine clearance is currently reduced and his white blood cell count is critically low. What to do next? a. Stop IV forscarnet and either give Intravitreal ganciclovir or forscarnet by injection, or use a sustained-release ganciclovir device intravitreally b. Repeat induction doses of foscarnet IV c. Continue forscarnet and induction doses of ganciclovir IV d. Add Zidavudine (AZT) to the current foscarnet therapy e. Stop foscarnet AAO 12 hal 193

35. A 30 years old man came to emergency. From examination found proptosis, chemosis, restriction of ocular motility, pain on movement of the globe, and fever. This patient diagnosed with orbital cellulitis that can be caused by. 1. dental infection 2. dacryoadenitis 3. trauma 4. endopthalmitis AAO 7 hal 44 36. A 60 old man was cared in eye department with locally invasive tumor. He had surgery, the eye and adjacent

intraorbital tissue are removed to locally excise the lesion, leaving periorbita and part of eyelid. What kind of type exenteration is it ? a. Extended b. Total c. Subtotal d. Perifer e. Partial AAO 7 hal 129

37. Parents of 4 year girl complain of their daugter eyes with the occurrence of exodeviation with fatique / stress, illness, day dreaming, drowsiness, or being scolded. Examination : exotropia manifest after fusion was disrupted by cover testing, and the deviation was remain greater at distance than at near after a period of monocular occlusion. We can classified this patient into.. a. True divergence excess b. Simulated divergence excess c. Convergence insufficiency d. Convergence paralysis e. Convergence access 38. What is diagnosis of that patient a. Intermitten exotropia b. constant exotropia c. convergence insufficiency d. convergence paralysis e. exophoria AAO 6 p. 105 - 114 AAO 6 p. 105 114

39. A 35 years old woman complain a pain on her right eyes and blurred vision. Blurred vision happed poor immediately with severe cephalgia. She came to ophthalmologist. She wa told that she got acute glaucoma and need some medicine. Normally humour aqueous is produced by: A. Pigmented epithelium layer of cilliary body B. Nonpigmented epithelium layer of cilliary body C. Stroma of cilliary body D. Muscle layer of cilliary body E. Connective layer of cilliary body AA0 10. P 17

40. A 4 years girl diagnosed with dense congenital cataract in the right eye, demonstrate exotropia 20 PD. The lens showed opacification of the Y-sutures of the fetal nucleus. These opacities often have branches or knobs projecting from them. Visual acuity appears normal.what is the variety of morphologic configuration of congenital cataract? A. Sutural B. Cerulean C. Coronary D. Capsular E. Anterior polar

Das könnte Ihnen auch gefallen